Eercicios de Estadistica 2

March 27, 2018 | Author: Wilmar Jose Gonzalez Palomino | Category: Type I And Type Ii Errors, Statistical Hypothesis Testing, Hypothesis, Sampling (Statistics), Variance


Comments



Description

CAPÍTULO VIII.CONTRASTES DE HIPÓTESIS 8.1 INTRODUCCIÓN Hipótesis: Enunciado acerca de una población, elaborada con el propósito de ponerse a prueba. Ejemplos de hipótesis acerca de un parámetro de población son: La media mensual de ingresos para analistas de sistemas es 2000, El 20% de los delincuentes juveniles son capturados y sentenciados a prisión Contrastar una hipótesis (Prueba de hipótesis) es comparar las predicciones con la realidad que observamos. Si dentro del margen de error que nos permitimos admitir, hay coincidencia, aceptaremos la hipótesis y en caso contrario la rechazaremos. Hipótesis nula: Es aquella hipótesis que se desea contrastar, se simboliza por Ho. Esta suele ser una estrategia o medio del que se sirve el investigador para probar la alternativa. El planteamiento de Ho permite elaborar un modelo probabilístico a partir del cual se puede llegar a una decisión final. Hipótesis alternativa: También se conoce como experimental y se representa por H1 o Ha. Esta es la hipótesis de investigación. De modo que se espera que hay un argumento para la hipótesis de investigación (o alternativa) H1, demostrando que no lo hay para su contraria, la hipótesis nula. Los contrastes pueden ser unilaterales o bilaterales (también llamados de una o dos colas) El contraste es bilateral (dos colas) si la hipótesis alternativa H 1 es del tipo ≠. El contraste es unilateral (una cola) si la hipótesis alternativa H 1 es del tipo < o >. Nota: generalmente una a hipótesis de investigación se plantea como una hipótesis alternativa, es decir que las hipótesis alternativa e hipótesis nula deben formularse de manera que al rechazar Ho, se apoye la conclusión de la investigación. 8.1.1 ERRORES DE TIPO I Y DE TIPO II. Si rechazamos una hipótesis cuando debiera ser aceptada, diremos que se ha cometido un error de tipo I. Error tipo I: Ho es cierto pero lo rechazo;  = P(rechazar Ho) cuando es cierta Error de tipo II: Es el error que consiste en no rechazar H 0 cuando es falsa. La probabilidad de cometer este error la denotamos con la letra β Error tipo II: Ho es falso pero lo acepto; β = P(aceptar Ho) cuando es falsa Errores y conclusiones correctas en las pruebas de hipótesis Se Acepta Ho Conclusión Se rechaza Ho Situación en la población Ho Verdadera H1 Verdeara Conclusión Error correcta tipo II Error tipo I Conclusión correcta Observaciones. 1. Los errores de tipo I y II no están relacionados más que del siguiente modo: Cuando α decrece β crece. Por tanto no es posible encontrar test que hagan tan pequeños como queramos ambos errores simultáneamente. De este modo es siempre necesario privilegiar a una de las hipótesis, de manera que no será rechazada, a menos que su falsedad se haga muy evidente. En los contrastes, la hipótesis privilegiada es H0 que sólo será rechazada cuando la evidencia de su falsedad supere el umbral del 100(1 − α) %. 2. Al tomar α muy pequeño tendremos que β se puede aproximar a uno. Lo ideal a la hora de definir un test es encontrar un compromiso satisfactorio entre α y β (aunque siempre a favor de H0). Denominamos potencia de un contraste a la cantidad 1 − β. 3. En el momento de elegir una hipótesis privilegiada podemos en principio dudar entre si elegir una dada o bien su contraria. 8.1.2 PRINCIPALES CONCEPTOS IMPLICADOS EN LA PRUEBA DE HIPÓTESIS: Nivel de significancia. Es la probabilidad de cometer un error de tipo I, cuando la hipótesis nula es verdadera, se denota con la letra α, y los mas conocidos son 0.05; 0.01; 0.1 El nivel de significancia se define como la máxima probabilidad de rechazar Ho cuando ésta es verdadera. Región crítica. El conjunto de todos los valores de la estadística de prueba que nos harían rechazar la hipótesis nula. Región de aceptación. Es la región complementaria de la anterior. Si el valor evaluado del estadístico pertenece a ella No rechazamos la hipótesis. (Las hipótesis nunca se aceptan de forma definitiva, sólo se aceptan provisionalmente, es decir, no se rechazan, a la espera de una nueva información que eventualmente pueda llevarnos a rechazarla en el futuro). Valor crítico: El valor o valores que separan la región crítica de los valores de la estadística de prueba que no nos harían rechazar la hipótesis nula. Los valores críticos dependen de la naturaleza de la hipótesis nula, la distribución de muestreo pertinente y el nivel de significancia α. En una prueba de dos colas, el nivel de significancia se divide equitativamente entre las dos colas. En una prueba de una cola, este nivel es el área de la región partir del valor crítico hasta el extremo derecho o izquierdo, según corresponda. α Región de aceptación Región crítica Región Crítica Estadística de prueba: Es una estadística obtenida de una muestra o un valor basado en datos de muestra. p-valor. El valor de  más pequeño que nos lleve a rechazar H0 se llama el p-valor de la prueba.. 8.1.3 PASOS DE LA PRUEBA DE HIPÓTESIS Pasos 1. Expresar la hipótesis nula y la hipótesis alternativa Ho: µ = µo Vs H1: µ ≠ µo o H1: µ < µo o H1: µ < µo Ejemplo Ho: µ = 16 H1: µ ≠ 16 2. Determinar y calcular una estadística de prueba z teo  x u s  16 ,12  16 n 0 ,5  1, 44 36 Para un α = 0,05; 3. Establecer los valores críticos que determinan las regiones de rechazo de las de no rechazo en función del nivel de significancia 4. Formular una regla de decisión Si Z < Z α /2 se rechaza Ho Si Z> Z α /2 se acepta Ho 5. Aplicar la regla de decisión (conclusión). 8.1.4 POTENCIA DE UNA PRUEBA DE HIPÓTESIS H0 se rechaza si z < – 1,96 o z > 1,96 No se rechaza H0 porque 1,44 es menor que el valor crítico 1.96 a manejando el número de repeticiones).30 (el ausentismo es menor del 30%) También se puede expresar Ho: Po = 0.30 (el ausentismo como mínimo es del 30%) H1: Po < 0.05 (nivel de significancia) n= 100 (tamaño de la muestra) Hipótesis Ho: Po ≥ 0. b) la varianza de la variable aleatoria y c) el tamaño de la muestra. siempre y cuando la vida media de una muestra de 110 tubos fluorescentes sea mayor a 1610 horas. La potencia se denota como π. 8. Con un nivel de significancia de 0. Se desea adquirir tubos fluorescente de una empresa. Cuanto mayor es la potencia mejor es la prueba.β. se puede afirmar que el jurado de elecciones tiene razón.1. Se elige una muestra de 100 individuos y se encuentra que 40 están dispuestos a votar. Se define como potencia a la probabilidad de rechazar la Hipótesis Nula cuando ésta es falsa.5 EJERCICIOS RESUELTOS En los siguientes ejercicios identifique los datos y plantee las Hipótesis Ejercicio 1. Un jurado de elecciones de cierto país dice que el porcentaje de ausentismo generalmente es de 30% como mínimo. Datos Po= 0. donde β es la probabilidad de cometer el Error de Tipo II.30 (el ausentismo como mínimo es del 30%) (Se considera solo el signo =) H1: Po < 0.05 se quiere saber si la duración media de los tubos es mayor de 1650 horas.Potencia de una prueba.60= 60% (porcentaje de ausentismo de la muestra)  = 0. La potencia es función de varios factores: a) el nivel de significación elegido. La potencia se calcula como π = 1 . Con un nivel de significancia del 5%. Solución. Esta probabilidad representa la chance de concluir que Ho es falsa cuando efectivamente lo es.30=30% (porcentaje de ausentismo poblacional) p = 60/100=0. Cuando el nivel de significación se ha fijado y la varianza de la variable aleatoria es conocida (o se ha estimado) es posible controlar la potencia de la prueba manejando el tamaño muestral (o. a) Dé las hipótesis nula y alternativa adecuada b) En esta situación ¿Cuál es el error de tipo I? ¿Qué consecuencias tiene cometer este error? c) En esta situación ¿Cuál es el error de tipo II? ¿Qué consecuencias tiene cometer este error? Solución Datos μ = 1650 (media de la población) . con una desviación típica de 100 horas.30 (el ausentismo es menor del 30%) Ejercicio 2. en el caso de los diseños experimentales. Sabiendo que los sueldos en esa multinacional se distribuyen de forma aproximadamente normal. El gerente afirma que el sueldo medio de sus trabajadores está por encima de los 1500 y los sindicatos afirman que es de 1400. 1056.05 . En un muestreo realizado entre los empleados de una multinacional se eligieron al azar 15 empleados y se anotó su sueldo mensual obteniéndose los siguientes datos: 1285.  1457 2  15( x ) 2 15  1 Desviación típica de la muestra Hipótesis Ho: μ = 1400 (el sueldo medio de todo los trabajadores es 1400) H1: μ ≠ 1400 (el sueldo medio de todo los trabajadores NO es 1400) b) datos μ = 1500 (media de la población) x 1285  152  .. 1812. 956. 1152. 1358 y 1457. c) El error de tipo II es aceptar que la duración media es menor o igual a 1610 horas siendo esta falsa. 1553. las consecuencias es que se adquiere productos que no cumplen las horas establecidas. 1120. 1423.  1457 15 media de la muestra =0. 1163.x  1610 a) Media de la muestra s= 100 desviación típica de la muestra =0. 1120. 1660..  1457 15 media de la muestra s 1285 2  1152 2  . Ejercicio 3. a) Crees que efectivamente el sueldo medio de todos los trabajadores es de 1400? b) Crees que lo que dice el gerente es cierto? c) En la situación del gerente ¿Cuál es el error de tipo I? ¿Qué consecuencias tiene cometer este error? d) En la situación del gerente ¿Cuál es el error de tipo II? ¿Qué consecuencias tiene cometer este error? Solución a) datos μ = 1400 (media de la población) x 1285  152  .. 1546. 1250.05 Hipótesis Ho: μ ≤ 1650 (No se adquiere los tubos) H1: μ > 1650 (Se adquiere los tubos) b) El error de tipo I es rechazar que la duración media es menor o igual a 1610 horas siendo estas verdaderas.... las consecuencias es que se deja de comprar focos que cumplen las horas requeridas. 33 (mas de 1/3 de las llamadas son urgentes con peligro de muerte) 8.1.333 (Proporción de la población) p= 61/150 (proporción de la muestra) Hipótesis Ho: Po = 0. Para comprobar si más un tercio de las llamadas a un servicio de ambulancias son urgencias con peligro de muerte.3 cuando la verdadera esperanza del nuevo proceso es µ= 14. se prueban 10 máquinas nuevas y se observa que éstas producen en promedio 13. el gerente miente) H1: μ > 1500 (el sueldo medio de todo los trabajadores esta por encima de 1500. b) ¿Cuál es el error de tipo I? ¿Qué consecuencias tiene cometer este error? c) ¿Cuál es el error de tipo II? ¿Qué consecuencias tiene cometer este error? . las consecuencias es que se le cree a un mentiroso.3 unidades.33 (No mas de 1/3 de las llamadas son urgentes con peligro de muerte) H1: Po> 0.3 vs. Se sugiere un nuevo proceso que es costoso de instalar. a) Calcular la probabilidad del error de tipo II en la prueba para µ= 12. Un proceso de fabricación produce 12. Para decidir si se hace el cambio o no.  1457 2  15( x ) 2 15  1 desviación típica de la muestra =0. Trabajar con α= 0. Se acepta que después de 3 años de almacenamiento el vigor de un arbusto forrajero medido como peso seco alcanzado a los 20 días de la germinación es de 45 mg promedio.05 Hipótesis Ho: μ = 1500 (el sueldo medio de todo los trabajadores No esta por encima de 1500. Ejercicio 2. las consecuencias es que se deja de creer a alguien que dice la verdad. Ejercicio 4. se ha tomado una muestra aleatoria de sus archivos y se ha encontrado que 61 de 150 llamadas son de este tipo. el gerente dice la verdad) c) El error de tipo I es rechazar que el gerente miente a pesar que realmente esta mintiendo.3 unidades por hora. obteniéndose los siguientes resultados de peso seco promedio a los 20 días: 49 43 56 57 59 65 52 51 50 55 60 65 53 57 67 56 53 37 45 42 a) Plantear las hipótesis nula y alternativa asociadas al problema.01.. Esta producción tiene una varianza igual a 4.6 EJERCICIOS PROPUESTOS Ejercicio 1.s 1285 2  1152 2  . Un nuevo método de almacenamiento se propone para aumentar el vigor. µ>12.. ¿Tiene fundamento dicha afirmación? Solución Po= 1/3=0. pero se piensa que puede incrementar la producción. Se evalúan para ello 20 lotes de 10 semillas cada uno y al cabo de 3 años se las hace germinar. d) El error de tipo II es aceptar que el gerente miente a pesar que dice la verdad. a. Se experimenta una nueva droga en 80 casos.Ejercicio 3. Señale el nivel de significación. . c. Un cierto tipo de cáncer tiene habitualmente una letalidad (número de muertos por cada cien enfermos) de 30. Señale la hipótesis de trabajo. Realice la prueba de significación estadística. b. en los cuales se producen 15 defunciones. la distribución de ~ N(μ.Xn una muestra aleatoria simple (m.1.1 PRUEBA DE HIPÓTESIS PARA LA MEDIA : MUESTRAS GRANDES.1. PRUEBA DE HIPÓTESIS PARA LA MEDIA DE LA POBLACIÓN.2 SUPUESTOS 1.a. la distribución cumple el teorema del limite central) con media desconocida µ.... no importa cómo sea la distribución de la población original. X2. Las medias de las muestras se distribuyen normalmente (distribución normal z). 9.1). . Es decir: Hipótesis Caso I Caso II Caso III Ho :  = 0 H1 :  < 0 Ho :  =0 H1 :  ≠ 0 Ho :  =0 H1 :  >0 9.. y 2 n x 2 con una varianza σ . ) y la variable z ~ N(0. 9. Deseamos contrastar la hipótesis de que el parámetro poblacional μ toma un determinado valor μ 0.CAPÍTULO IX. El tamaño de la muestra es grande n≥30 2. X3.1 INTRODUCCIÓN Sean X1. Las observaciones son independientes 3.s) de una distribución normal o cualquier distribución (si n≥30. es la distribución normal estándar N (0.).1). cuya media es µ y varianza finita σ 2. a) Se conoce la varianza poblacional σ2. entonces la forma límite de la distribución de: z x  n conforme n →∞. s        x  Z  / 2    x  Z / 2  n  n n n x Donde: μ : media poblacional n : Tamaño de la muestra  : Desviación estándar poblacional : Media de la muestra s : Desviación estándar de la muestra x Nota: para encontrar s se utiliza la siguiente formula s 2 2  x  nx n 1 x Teorema del límite central: Si es la media de una muestra aleatoria de tamaño n que se toma de una población con cualquier distribución (oblicua a la derecha. Z x  uo x  uo Z s  n n Intervalo de Confianza del 100(1-)% para la media poblacional .. es de la forma: a) Se conoce la varianza poblacional σ2.  s  n  x  Z  / 2    x  Z / 2 b) Se desconoce la varianza poblacional σ 2. . con forma de tina.Estadístico de prueba b) Se desconoce la varianza poblacional 2.. oblicua a la izquierda. etc. 42 172 Paso 3.42 . la gerente de crédito. 38.9.05=1. Carla.3 EJERCICIOS RESUELTOS Ejercicio 1. con un nivel de significancia del 5%?. Una revisión al azar de 172 saldos reveló que la media de la muestra es S/.400.645 Z=2.645 Paso 4.A emite su propia tarjeta de crédito.1.42) > (z0.05. quiere encontrar si la media mensual de saldos no pagados es mayor que S/. Decisión H0 se rechaza si Z=2. Solución Datos.645 Zona de Aceptación Ho Zona de rechazo Ho Z0. Establecemos la región de rechazo para un =0.05=1.400.407 y la desviación estándar de la muestra es S/. de la tabla normal z0.05=1.645) Zona de Aceptación HoZona de rechazo Ho Z0.05 (nivel de significancia 5%) s = 38 n = 172 Paso 1: Establecemos las hipótesis H0 : μ = 400 (La media mensual de saldo no pagados no es mayor de 400) H1 : μ > 400 (La media mensual de saldo no pagados si es mayor de 400) Nota: la Ho también puede expresarse como Ho : μ ≤ 50 Paso 2: Calculamos el estadístico de prueba Como n > 30 la prueba es la distribución normal Z z x u s n  407  400 38  2. La casa Carma S. ¿Debe Carla concluir que la población media es mayor que S/.42 cae en la zona de rechazo (zona achurada) es decir si (z=2.05=1. x  407 μ = 400 = 0. 400.645). SOLUCIÓN EN MINITAB Ingresamos al minitab y hacemos click en Stat>Basic statistics>1-sample Z.42) cae en la zona de rechazo. o lo que es lo mismo (Z=2. rechazamos la Ho.... como se ve en la figura siguiente Tamaño de la muestra ( n ) Hipótesis alternativa (mayor que “ > “ ) la po b la ci ón () Nivel de confianza (1-) (s )o de Aparece la siguiente pantalla. luego ingresamos los datos en esta.05=1.42)> (Z 0. Es decir rechazamos que (La media mensual de saldo no pagado no es mayor de 400) Luego Carla puede concluir que hay evidencia suficiente para aceptar que la media de saldos no pagados es mayor que S/. D es vi ac ió n es tá nd ar de la m ue st ra Media de la población ( μ ) Media de la muestra ( n ) .Paso 5: Conclusión Como (Z=2. 008. Para salir de dudas.234 (valor mínimo del nivel de confianza al 95% ) Z= 2.15 kilogramos.42 ( z calculado) P = 0. El peso de un grupo de niños debe ser de 50.01 s = 5.2 α = 0. luego OK y los resultados son: One-Sample Z Test of mu = 400 vs > 400 The assumed standard deviation = 38 95% Lower N Mean SE Mean Bound Z 172 407.234 2.Hacemos click en Options… ingresamos el nivel de confianza y elegimos la Alternative.15 Prueba de hipótesis Ho: μ = 50 (los niños NO tienen pesos menores que 50kg) H1: μ < 50 (los niños tienen pesos menores a 50kg) Nota: la Ho también puede expresarse como Ho : μ ≥ 50 .20 kilogramos.008 (de 100 veces que rechazamos Ho por ser falsa. hay un probabilidad de 0. finalmente presionamos OK. ¿puede afirmarse que el peso de todos los niños es menor de 50. los trabajadores de salud afirman que tienen un peso menor a 50.42 o en función de P=0. Sin embargo. Datos μ = 50 n = 36 x  49. Como (P = 0. con una desviación estándar de 5.8% de equivocarse) Decisión: la decisión se puede tomar en función de Z=2.42 P 0.008 Interpretación: N=172 (tamaño de la muestra ) Mean= 407.897 402.000 2.008)< (α =0. Ejercicio 2. obteniendo un peso de 49. se tomó una muestra de 36 niños.00 kilogramos? Solución.00 (media de la muestra) SE Mean = 2.05) entonces se rechaza la Ho Conclusión.00 kilogramos. Carla tiene evidencia suficiente para aceptar que la media de saldos no pagados es mayor que S/. Con un 99% de confiabilidad.400.897 (error estándar de la media) Lower Bound = 402.00 kilogramos. En el segundo caso. y se obtiene la figura siguiente.0.Z x  u 49.33 de tabla normal Z0. reemplazando datos Para decidir si rechazamos o aceptamos la Ho Construimos el gráfico y determinamos Z0.932 esta dentro de la zona de aceptación.01 = – 2.15 n 36 Estadística de prueba Como n>30 utilizamos la prueba Z.932 s 5. Utilizando Minitab Ingresamos al minitab y hacemos click en Stat>Basic statistics>1-sample Z.932 Conclusión. Aquí ingresamos los datos y activamos Options…. para ingresar el 1- .2  50   0.. Como Z= – 0..01=-2. los niños tienen pesos mayor o igual a 50. es decir.33 z=. entonces aceptamos Ho. 8583 50. reemplazando datos . es decir no hay suficiente evidencia para rechazar que los niños tengan pesos mayores o iguales a 50.667 ) 59  2.6118 -0...15 95% Upper N Mean SE Mean Bound Z P 36 49.176) es mayor que (= 0.05) entonces se acepta la Ho. Los invitados a una reunión de trabajo tienen una tolerancia de 5 minutos en promedio.00 kg.52020 Hipótesis Ho: μ = 5 min (los invitados están dentro de la tolerancia) H1: μ > 5 min (los invitados NO están dentro de la tolerancia) Nota: la Ho también puede expresarse como Ho : μ ≤ 5 min Estadística de prueba Como n>30 utilizamos la prueba Z. ¿Los invitados están fuera del tiempo de tolerancia?.93 0. los resultados fueron 2 6 7 5 9 5 5 0 7 5 1 1 7 9 0 6 5 2 3 5 5 4 6 3 5 2 3 7 6 8 1 1 7 8 4 2 8 4 2 7 6 7 5 2 9 2 1 6 6 4 9 4 6 3 4 8 4 6 2 9 A nivel del 10%.10 n = 60 x Calculamos la media aritmética ( x s 2  6  7  ..  6  2  9 60 ) y la desviación estándar de la muestra (s)  4.176 Conclusión: Como (P=0.2000 0.7667 2 2 2 2 2 2 2 ( 2  6  7  . Solución Datos μ=5  = 10% = 0. para lo cual se midió el tiempo de demora de 60 invitados. Se quiere saber si un grupo de invitados están dentro de la tolerancia.Presionamos Ok luego Ok y los resultados obtenidos son: One-Sample Z Test of mu = 50 vs < 50 The assumed standard deviation = 5.  6  2  9 )  60 ( 4.. Ejercicio 3. .72 60 Para decidir si rechazamos o no la Ho Construimos el gráfico y determinamos Z0.10= 1.7667  5 2..72 Z0. esta no se rechaza. Utilizando MINITAB Ingresamos al minitab y hacemos click en Stat>Basic statistics>1-sample Z.1 = 1.Z  x u s n  4.28 Conclusión. como se ve en la figura siguiente .72 cae en la zona de aceptación de Ho.28 de tabla normal Zona de Aceptación HoZona de rechazo Ho Z=-0. Como z=-0. por lo tanto los invitados están dentro del tiempo tolerado.52  0. 763>=0.01? Datos μ = 500g Para determinar la media y la desviación estándar utilizamos Minitab. verifica que la presentación del producto en bolsas de 500 g.1) se acepta Ho. Para esto utilizamos Stat>Basic statistics>Display Descriptive estatistics… Finalmente los resultados de la prueba de hipótesis son One-Sample Z: Tiempo Test of mu = 5 vs > 5 The assumed standard deviation = 2. con una media de 497 g. como (p=0.76667 StDev 2. si el nivel de significancia es 0. ¿Podemos afirmar la hipótesis de que las bolsas contienen menos de 500 g.32533 90% Lower Bound 4. Ejercicio 4. y con 5% de nivel de significancia.52020 SE Mean 0.? Solución a) ¿Hay evidencia suficiente en base a esta muestra.Los datos lo ingresamos en la columna C1 (Tiempo) Nota: la desviación estándar de la muestra primero debemos encontrarlo. contenga dicha cantidad.72 P 0. de que el contenido de las bolsas es diferente a 500 g. Se obtuvo una muestra de 50 bolsas los cuales tuvieron los siguientes pesos en gramos: 505 500 494 495 409 498 514 497 500 498 500 507 503 596 504 501 515 506 510 499 495 499 497 508 501 597 489 499 496 497 508 506 498 523 522 502 499 506 498 503 497 495 485 497 511 500 500 497 519 498 a) ¿Hay evidencia suficiente en base a esta muestra.763 Conclusión.52 Variable Tiempo N 60 Mean 4. si el nivel de significancia es 0.34974 Z -0. esta de 25 bolsas. y una desviación de 10 g. El control de calidad de una cooperativa que produce azúcar. de que el contenido de las bolsas es diferente a 500 g. Por lo tanto los invitados están dentro del tiempo tolerado. de la siguiente manera Ingresamos los datos en C1 como se ve en la figura y Hacemos clic en Stat>Basic statistics>Display Descriptive estatistics… .01? b) Se obtuvo otra muestra. finalmente hacemos click en OK.00 Median 500.01 tenemos un nivel de confianza de 99% Utilizando minitab se tiene Q3 506.00 Q1 497. el resultado es Descriptive Statistics: C1 Variable C1 Variable C1 N N* Mean 50 0 503.45 StDev 24.00 Interpretación N=50 (tamaño de la muestra) x Mean = 503.25 (tercer cuartil) Hipótesis Ho: μ = 500 g (el contenido es igual a 500g) H1: μ ≠ 500 g (el contenido es diferente a 500g) Para un nivel de significancia de 0.00 (Me Mediana) Q3=506.00 (primer cuartil ) Median=500.86 Maximum 597.00 SE Mean 3.36 (s Desviación estándar de la muestra) Q1=497.36 Minimum 409.86 ( media de la muestra) StDev =24.25 . ¿Podemos afirmar la hipótesis de que las bolsas contienen menos de 500 g. es decir el contenido es igual a 500g b.263 .05.360 3.12 Conclusión Como (p=0. 512.  =5% = 0.? datos μ= 500 n = 25 bolsas x  497 g s = 10 g.36 Variable N Mean StDev SE Mean C1 50 503.734) Z 1.95 Hipótesis Ho: μ = 500 g (el contenido no es menos de 500g) H1: μ < 500 g (el contenido es menos de 500g) Utilizando minitab se obtiene One-Sample Z Test of mu = 500 vs < 500 The assumed standard deviation = 10 P 0.445 99% CI (494.263) > (=0.01) se acepta Ho. osea que 1. = 0.986.860 24.Finalmente hacemos clic on Ok luego en OK Los resultados son One-Sample Z: C1 Test of mu = 500 vs not = 500 The assumed standard deviation = 24. se tomó una muestra de 50 habitantes y se encontró que tenían una media de 60. No se rechaza la Ho.05).067 Conclusión Como (P=0. =0.000 95% Upper Bound 500. es decir el contenido no es menos de 500g. Hacemos uso de MINITAB One-Sample Z Test of mu = 65 vs > 65 The assumed standard deviation = 1.00 1.50 P 0.05) se acepta Ho.067) > (=0. o sea que no hay evidencia que la edad promedio de la ciudad sea mayor de 65 año .N 25 Mean 497. Ejercicio 5.6710 -25.0000 0. Calcular el valor de P para el ensayo de hipótesis en donde se quiere probar que la edad promedio de los habitantes de una ciudad es superior a 65 años.05.4142 95% Lower N Mean SE Mean Bound Z P 50 60.000 SE Mean 2. una varianza es 2 años a un nivel de significancia de 0.000 DECISIÓN: Como (P=1)> (α=0.05 Hipótesis Ho: u ≤ 65 Ho: u > 65 (Edad promedio de la ciudad no es mayor a 65) (Edad promedio de la ciudad es mayor a 65).2000 59. Solución Datos μ = 65 n = 50 x  60 Sha = 2.290 Z -1. nos dicen que la variable tiempo transcurrido entre la administración del antiinflamatorio y el momento en que desaparece la inflamación sigue una distribución normal de media 14 y desviación 7.03 18 Hipótesis Ho: μ =14 H1: μ >14 Z0.645 Z=10 . Se sabe que la media y desviación típica normal de la población es respectivamente 110 y 20 mmHg.05=1. Solución: x  19 Datos.05 se rechaza la Ho. Se pide comprobar la afirmación del laboratorio a un nivel de significación de 0. obteniéndose una media muestral de 130 mmHg. σ= 7. Además.05? Solución Datos x  130 mmHg n = 100 μ = 110 mmHg  = 20 mmHg Hipótesis Ho: μ = 110 (no sufren de hipertensión) H1: μ > 110 (sufren de hipertensión) z x u  n  130  110 20 100  10 .645.05. μ = 14.  z x u   n . NOTA: A pesar que (n=18) < 30 utilizamos la distribución Z y no la t esto se debe a que conocemos la desviación típica de la población σ =7 Ejercicio 7. Con la finalidad de comprobar estadísticamente esta afirmación.05=1. a los que se ha medido la presión sistólica. El tiempo medio de respuesta de la muestra fue de 19 minutos. Luego como Z>Z0. se elige al azar 18 cerdas con inflamaciones varias y se toma como variable de respuesta el tiempo transcurrido entre la administración del antiinflamatorio y el momento en que desaparece la inflamación. Se ha tomado una muestra aleatoria de 100 individuos para determinar si tienen hipertensión. Un laboratorio farmacéutico afirma que el antiinflamatorio de su fabricación elimina la inflamación en 14 minutos en los casos corrientes. n = 18 19  14 7  3. ¿Cual es su conclusión a un nivel de significancia de 0.Ejercicio 6.00 Zona de Aceptación HoZona de rechazo Ho Z0. 9 años.4 n 1. dieron una media de 2 cm y una desviación típica de 0.44% 99. 95.4 años s = 1. una muestra de 100 y se encuentra que tienen una edad promedio de 19. De los alumnos anteriores se elige.9  5.4 Ejercicio 9.1 cm.4) z x u   19 .05=1.5  18 .26% b. 68.4 años.025=1.73% Para el diámetro de todos los cojinetes.5 años n = 100 μ = 18.645 Del gráfico concluimos que se rechaza la Ho. 99. Ejercicio 8.79 Z0. Hallar los intervalos de confianza del: a. Las medidas de los diámetros de una muestra tomada al azar. hechos por una determinada máquina.05.Z0.9 años Hipótesis Ho: μ = 19. con un nivel de significancia de 0. De su opinión al respecto.73% -z/2 z/2 x1 x2 .96 Del gráfico concluimos que se rechaza la Ho. o sea la edad promedio es diferente a 18. de 200 cojinetes de bolas.025 =-1. Se cree que la edad promedio de los alumnos que ingresan a la Universidad es de 18.5 (la edad promedio es 18.4) H1: μ ≠ 19.5 años con desviación típica 1. al azar.96 Z=5.025 =1.44% c.26% 95.79 Zona rechazo Zona Ho de aceptación Zona rechazo Ho Ho 100 Z0. 68.5 (la edad promedio es diferente a 18.96 Z0. o sea los individuos sufren hipertensión. Solución Datos x  19 . 2.96 n  1. 0.1      . x2 . 2  2. de la tabla (dos colas) se tiene z/2=1. Luego la muestra debe ser 50 como mínimo .993 .9 si 1. x  Z / 2   x  Z / 2  200   n n   1.81 entonces =0.95 entonces =0.05.00    .= 0.007  0.00 . cuya varianza es conocida. ¿Cuál ha de ser el tamaño de la muestra? Solución Datos.25 = z x u  n Para =0.81.00 = c. 2.00 0.05.6826  0.1  200  2  3. 2=0. x  Z / 2   x  Z / 2  200   n n = 1-=0.25 con una confianza del 95%.79. admitiéndose un error máximo de 0. x  Z / 2   x  Z / 2  200   n n = = Ejercicio 10.00 .96 x 0.9973  1.1.9 0. s=0.00 .25 .986 . Se sabe que el contenido de lactosa de cierto alimento lácteo sigue una distribución normal.1      . teniendo un valor de 0.014   0. 2. 1-=0.1   = = b.25 0 . Se desea estimar el valor de la media poblacional mediante el valor de la media de una muestra.1  200  2  2. n=49. x e = 0. 1-=0.96 -μ  0. 2  1.9 n  1. a.1  200  2  1.Solución n = 200.9544 1.021   0. 2  3.979 . 05 que los pesos de los productos esta por debajo de 1.96 5      .35 kg. =0.15 z2 x2 x2= u+z2 =68+0.65 (cultura media). se observa que las puntuaciones siguen una distribución normal. Si se extrae una muestra aleatoria de 64 alumnos y para un nivel de significación del 5%.84(18)= 52.15 .15 9. se obtuvo una producción media de 45 y no se observó ningún cambio apreciable en la dispersión que estaba establecida en =1. En una fabrica de productos lácteos.20 (baja cultura). =0.05 z=1. la nota sigue una distribución normal N(14.84(18)=83.Ejercicio 11.5).4 EJERCICIOS PROPUESTOS Ejercicio 1. =18 μ =68 =0.5 kg? Ejercicio 2. con una desviación estándar de 0.96 . ¿Se debe efectuar el cambio tecnológico? . x  Z / 2   x  Z / 2  64   n n = = Ejercicio 12.15 (alta cultura) x1= u-z1 =68-0. el segundo un 65% y el tercero el 15% restante. Una empresa estudia introducir un nuevo sistema de producción para mejorar su productividad media establecida actualmente en 40 unidades por persona diaria.. ¿Se puede asegurar para un α = 0. aplicada a 40 personas.22   5  64  14  1.1. Se estima que el cambio no será rentable si no consigue elevar dicho numero por encima de 44 u. dieron en promedio un peso de 1. ¿en qué condiciones se rechazaría la hipótesis de que la media de la población es de 14? Solución Datos Para =0.96 (dos colas) La hipótesis se rechazaría cuando el valor de la media de la muestra no pertenezca al intervalo de confianza siguiente. de media 68 y desviación típica 18. Tras realizar un test de cultura general entre los habitantes de cierta población. de cultura general excelente). ¿Cuáles son las puntuaciones que marcan el paso de un grupo a otro? Solución Datos.2 kg.0 u. de cultura general aceptable.20 z1 x2 0. Realizada una prueba con la nueva tecnología. 14  1. En una determinada población estudiantil.  12.85 0. una muestra de 100 unidades. por día. de manera que el primer grupo abarque un 20% de la población.78 . Se desea clasificar a los habitantes en tres grupos (de baja cultura general.65 0. Contrastar la hipótesis de que μ=1650 h. se piensa que la desviación típica de la población es 90h. a un nivel de significación del 5%. ¿Se puede afirmar a nivel de significación 0.Ejercicio 3. Ejercicio 8. En estudios previos se ha determinado que el nivel de colesterol promedio de pacientes con problemas cardíacos es 222. Se sabe que el sueldo anual de los trabajadores de una empresa sigue una distribución normal de media desconocida y desviación típica de s/500. se toma una muestra de 36 taxis.05 que el tiempo de duración media de los productos es 100 días?. frente a la alternativa μ≠1650.05. frente a la alternativa μ >1650. Una encuesta realizada a 164 trabajadores de una fábrica. ¿Cuáles son las hipótesis nula y alternativa del contraste? b. b. ¿Se acepta la hipótesis nula con el nivel de significación indicado? Ejercicio 9.00 Para saber si es cierto.. a. Un cardiólogo piensa que en realidad el nivel es más alto y para probar su afirmación usa la muestra siguiente. ¿El salario medio es mayor o igual que 500 a un nivel de significación del 5%? Ejercicio 6. Ejercicio 11. si = 0. que opina de la afirmación de la policía. 217 223 234 215 245 238 248 220 217 225 226 240 226 216 221 230 202 199 217 214 233 224 210 210 235 236 205 242 218 222 219 215 220 Ejercicio 7. se eligió una muestra aleatoria de 60 artículos y se comprobó que su tiempo de duración fue de 80 días con una varianza de 16. Con un 95% de confiabilidad. Se ha observado el sueldo anual de 36 trabajadores de dicha empresa escogidos al azar.0 kg. El promotor afirma que en un mes los participantes bajaría 5kg.50 . Un profesor universitario está interesado en determinar si las computadoras mejoran el rendimiento de sus alumnos de estadística.Tiene razón el MTC al 95% de confianza. considere una población normal. y se consideran pruebas normalizadas. si la media de la distribución muestral es de s/2500. Con un 99% de confiabilidad. Se quiere probar la eficacia de una dieta para bajar de peso. Utilizando α= 0. luego de un mes se obtuvo una disminución de peso en promedio 4kg y una desviación estándar de 1. Ejercicio 4. c.5 con desviación estándar s/0. Con este propósito seleccionó aleatoriamente 35 estudiantes del tercer ciclo y después de facilitarles el uso de computadoras durante un semestre. resultando que el precio medio es de s/6. Determina la forma de la región crítica. ¿Habrá suficiente evidencia estadística para apoyar la afirmación del cardiólogo? Justificar su contestación. se encuentra una media de 1700h. Se quiere estudiar la vida media μ de unas resistencias producidos en una empresa. Al extraer una muestra de 80 resistencias. a. determine la conclusión a que llegó el profesor?. b. Por experimentos anteriores. determinó que las notas fueron en promedio 15 con varianza de 4. Ejercicio 5. ¿puede afirmarse que es cierto la afirmación del promotor?. Si los promedios de estadística en años anteriores eran de 12. Se hizo una encuesta a una muestra de 200 denunciantes cuyas denuncias fueron resueltas y se encontró que sus casos fueron resueltos en 15 día en promedio. 600 con una desviación típica de 64. Ídem. concluyó que el sueldo medio es de s/. y se ha obtenido un valor medio de s/. 2400. Ejercicio 10. En la etiqueta de un producto comestible figura que el tiempo de duración de este es de 100 días. a. 221 240 210 . La policía afirma que las denuncias por robo son resueltas en 10 días en promedio. El MTC afirma que los pasajes de taxi local son de s/5. Probar la hipótesis.01. con una varianza de 4. ¿puede afirmarse que es cierto la afirmación del promotor?. para comprobar se puso a dieta a 66 personas. El equipo afirma que la presión sistólica media en varones jóvenes estresados es mayor que 18 cm de Hg. Los trabajadores de una empresa que utilizan el autobús regular se quejan de que éstos se están retrasando en media más de un cuarto de hora. Ejercicio 14. El gerente afirma que el contenido promedio de los botellas envasadas por su representada es igual a 320ml. para α= 0. determinar si tiene razón el gerente.05. . a) ¿Se descartaría la hipótesis μ = 15 en favor de la alternativa μ ≠ 15. Ejercicio 15. ¿qué sucedería con un nivel de significación del 1%? d) Construir un intervalo de confianza del 99% para μ.01.6 . Suponga una desviación estándar poblacional de 8.05 y α = 0. 144) con μ desconocido. Para comprobarlo han anotado el retraso de algunos días elegidos al azar. Con un nivel de confianza de 95% . Una variable aleatoria sigue una distribución N(μ.05. ¿Se puede afirmar que el equipo tenía razón? Ejercicio 17.9 años. c) Considerando la misma hipótesis del punto a). si una muestra aleatoria de n = 64 observaciones arroja una media igual a 20? b) Construir un intervalo de confianza del 95% para μ. Plantee y resuelva como mínimo 4 ejercicios. Los datos fueron: 12 23 11 24 4 36 15 16 16 3 17 25 17 6 11 15 35 8 14 22 19 17 4 9 14 25 16 12 14 26 5 15 18 26 12 10 11 4 3 42 17 13 21 14 22 5 7 7 8 11 12 23 16 24 19 11 2 17 6 20 17 25 25 24 23 9 14 26 42 16 6 16 7 24 13 15 3 13 6 6 22 25 24 4 6 35 9 6 7 14 5 23 8 12 18 14 26 23 7 5 A nivel de significación del 10 %. ¿esto parece indicar que la vida media hoy en día es mayor que 70 años? Utilice un nivel de significancia de 0. Comparar con los resultados obtenidos en los puntos a) y c). de prueba de hipótesis tomando como datos problemas de su área. Un equipo de psicólogos está estudiando el efecto del estrés sobre la presión arterial en alumnos de la UNHEVAL. ¿crees que tienen razón los trabajadores? Ejercicio 16. e) Probar H0: μ = 15 versus H1: μ > 15 para α = 0. Una muestra aleatoria de 100 muertes registradas en un país el año pasado muestra una vida promedio de 71.8 años. Para confirmar lo dicho por el gerente. y encuentra que la media es 300 ml.Ejercicio 12. Ejercicio 13. Se toma una muestra de 36 alumnos y se encuentra que x  18. Suponga que los datos provienen de una población aproximadamente normal.5 s  3. se toma una muestra de tamaño 10. 9.2. X n el supuesto de normalidad en las muestras. Estadístico de prueba. s        x  Z  / 2    x  Z / 2  n  n n . tiende a una distribución t Se quiere probar las hipótesis Hipótesis Caso I Ho :  = 0 H1 :  < 0 Caso II Caso III Ho :  =0 H1 :  ≠ 0 Ho :  =0 H1 :  >0 9.1 INTRODUCCIÓN Para realizar pruebas de hipótesis acerca de la media el tamaño de la muestra es pequeño. L .2 PRUEBA DE HIPÓTESIS PARA LA MEDIA POBLACIONAL: MUESTRAS PEQUEÑAS 9. Se conoce la 2 poblacional (pocas veces sucede) t b) Se desconoce la 2 poblacional x  uo x  uo z s  n n ~ t n -1 Intervalo de Confianza. La muestra es pequeña (n < 30) 3.2 SUPUESTOS 1. Supongamos que T es una muestra x s n aleatoria de una población normal con media μ y varianza σ.  s  n  x  t ( / 2. del 100(1-)% para la media poblacional . 2. a). Los valores de la muestra provienen de una población con una distribución normal o aproximadamente normal. X 2 . n 1)    x  t ( / 2. la con n-1 grados de libertad.2. n 1) b) Se desconoce la varianza poblacional σ 2. es necesario X1 . es de la forma: a) Se conoce la varianza poblacional σ2. Las muestras son muestras aleatorias simples. 05. Como la población es normal y n<30 se usa la tabla de una cola t (α.n x Donde: n : Tamaño de la muestra x : Media de la muestra μ : Media de la población  : Desviación estándar poblacional s : desviación estándar de la muestra n-1 : grados de libertad Nota: para encontrar la media y la desviación estándar se utiliza las siguientes formulas n x  xi i 1 n s 2 2  x  nx n 1 Nota: para determinar si la distribución es normal. Datos n =16. para de las vegetarianas Solución.15) = 2. Con una desviación típica de 5kg.3 EJERCICIOS RESUELTOS Ejercicio 1. Considerar distribución normal.n-1)= t (0.05. El peso medio en mujeres comprendidas entre 30 y 40 años. Un estudio realizado en 16 mujeres con edades comprendidas en el intervalo anterior y que siguen una dieta vegetariana da un peso medio de 45kg. a. Puede considerarse que la dieta vegetariana produce una modificación del peso medio de las mujeres?. b.131 t=8 . a) Puede considerarse que la dieta vegetariana produce una modificación del peso medio de las mujeres?.15) =2.131 t(0. Dar un intervalo de confianza al 95%.131 Zona rechazo Zona Ho de aceptación Zona rechazo Ho Ho t(0.05.2. Hipótesis Ho: μ = 35 (La dieta vegetariana NO produce modificaciones en el peso de las mujeres) H1: μ ≠ 35 (La dieta vegetariana SI produce modificaciones en el peso de las mujeres). shapiro-Will o Kolmogorov-Smirnov ver ejercicio 3 9.05. es de 35kg. podemos utilizar las pruebas de Andersondarlin.15) =-2. α=0. o sea la dieta si produce modificaciones b.0000 StDev 5. 47. 47.3357. se rechaza Ho.6643 16  Solución en MINITAB Ingresamos al minitab y hacemos click en Stat >Basic statistics >1-sample t.131).t  x  u 45  35 10   8 s 5 5 n 16 4 Si t > t (0.n 1 s   n .15) = 2.6643) T 8.131 5     42.05. INTERVALO:  s  n  x  t ( .0000 SE Mean 1. como se ve en la figura siguiente Los resultados son los siguientes One-Sample T Test of mu = 35 vs not = 35 N 16 Mean 45. 45  2.05..000 .n 1)  5  16  45  2.3357.15) = se rechaza la Ho Decisión: como (t=8) > (t (0. x  t ( .00 P 0.131 .2500 95% CI (42. 1812.. 1546.. 1553.6643) al 95% Ejercicio 2.  1358 2  1457 2 )  15(1327. 1358 y 1457. 1120. 956. 1152...DECISIÓN: Como (P = 0.4 15 (1285 2  1152 2  1546 2  . INTERVALO: (42. 1423.000) < (α = 0.. Crees que lo que dice el gerente es cierto? Solución. En un muestreo realizado entre los empleados de una multinacional se eligieron al azar 15 empleados y se anotó su sueldo mensual obteniéndose los siguientes datos: 1285.3357. Sabiendo que los sueldos en esa multinacional se distribuyen de forma aproximadamente normal. 1056.105 Prueba de Hipótesis Ho: μ=1500 (sueldo promedio no es mayor que 1500) H1: μ>1500 (sueldo promedio es mayor a 1500..4) 2 15  1 s = 244. 47. 1163.. 1120. utilizamos la prueba t de student x  u 1327.... El gerente afirma que el sueldo medio de sus trabajadores está por encima de los 1500 y los sindicatos afirman que es de 1400. 1660. 1250. Datos x  ¿? n = 15 μ=1500 (sueldo promedio según el gerente) s = ¿? x Primero hallamos s y x x s 1285  1152  1546  1423  1120  1660  956  1250  1812  1120  1553  1056  1163  1358  1457 15 19911  1327. según el gerente) Estadístico de prueba Como la población es aproximadamente normal y n<30 .4  1500 t   2..738 s 244.05) luego se rechaza Ho..105 n 15 . 05.0. de la tabla. .761 Zona de rechazo Ho Zona de Aceptación Ho t = – 2.739 cae en la zona de aceptación aceptamos Ho. Ejercicio 3.05 Primero Dado que no sabemos si la población es normal. Un fabricante de lámparas eléctricas sostiene que la duración media de las mismas (horas) es en promedio superior a 1300 h. entontes debemos determinar si la distribución es normal. Solución Datos μ= 1300 n =16 x ? s=?  = 0.734 t(14.0. es decir que el gerente esta equivocado.05)=1. para eso aplicamos la prueba de Kolmogorov-Smirnov de Minitab Ho: la distribución es normal H1: la distribución no es normal Si P-value es mayor que 0.05) =1. la distribución se considera normal. Como t=–2. el sueldo de los trabajadores no es mayor que 1500. Hacemos click en >Stat>Basic Statistics>Normality Test….Para decidir si rechazamos Ho o aceptamos Ho. construimos la grafica y determinamos el valor de t. Se toma una muestra de 16 lámparas siendo el resultado de la inspección el siguiente: 980 1350 1020 1140 1520 1390 1205 1180 970 1420 1850 1300 1305 1040 1050 1520 Verificar la Ho del fabricante con un coeficiente de riesgo del 5%. t(14.761 Conclusión. 126 >0.3 16 0. para obtener los resultados Los resultados son Probability Plot of C3 Normal 99 Mean StDev N KS P-Value 95 90 Percent 80 70 60 50 40 30 20 10 5 1 600 800 1000 1200 1400 C3 1600 1800 2000 1265 241.Los datos lo ingresamos en la columna C3.150 . y hacemos click en OK. .Tenemos que p-Value puede tomar valores mayores a 0. t(0.. luego completamos los datos como se ve en la figura y clickeamos ok. Segundo Sabiendo que la distribución es normal para nuestro ejemplo aplicamos la prueba de t de estudents Hipótesis Ho: μ ≤ 1300 (La duración de las lámparas no es mayor que 1300) H1 : u >1300 (La duración de las lámparas es mayor que 1300) Utilizando MINITAB tenemos: Ingresamos las horas en C3. luego se acepta la hipótesis nula.16) = 1. es decir la distribución es normal.05).150 Para un nivel de significancia de 0.24 T -0.756.00 StDev 241. luego ejecutamos las opciones Stat/Basic statistics/1-sample t. ok Los resultados son: One-Sample T: C3 Test of mu = 1300 vs > 1300 Variable C3 N 16 Mean 1265.05.56 < 1. en el grafico puede confirmar esta afirmación.715 .33 95% Lower Bound 1159.15) > (=0.32 Decisión T= – 0.58 P 0.746 Luego 0. se acepta Ho.05 (p-value=0.58. SE Mean 60. 064 t(0. t  x  u 170  174  4    2 s 10 10 n 25 5 Zona Zona rechazo Hode aceptación Zona rechazo Ho t(0. Conocemos que las alturas X de los individuos de una ciudad.000 10.24)=-2. Es decir que la duración de las lámparas no es mayor que 1300 horas. Para ello nos basamos en un estudio en el que con una muestra de n = 25 personas se obtuvo: Solución: Como n< 30 se toma la prueba de t de student por ser la distribución gaussiana (normal) El contraste que se plantea es: H0: μ = 174 (la altura media es igual a 174) H1: μ ≠ 174 (la altura media es diferente a 174) La técnica a utilizar consiste en suponer que H0 es cierta y ver si el valor que toma el estadístico Cae dentro de la zona de rechazo de Ho.705)> (α=0.24) Ho =-2.05.24) =2.000 2. Como t= –2. cae dentro la zona de aceptación de Ho.128) T -2.05) luego se acepta Ho.05.05.057 Aquí su interpretación: Test of mu = 174 vs not = 174 Prueba de la media (Ho: μ = 174) vs (Ho: μ ≠ 174) N Mean StDev SE Mean 95% CI T P . Deseamos contrastar con un nivel de significación de α = 0.05. además (t= –2) < (t (0.064 Conclusión.00 P 0.También (p = 0.872.05. 174.24)=– 2.064 t= – 2 t(0.24)= 2.000 (165. es decir la altura media es igual a 174 En MINITAB teems: One-Sample T Test of mu = 174 vs not = 174 N Mean StDev SE Mean 95% CI 25 170.064t=-2 t(0. X  170cm S  10 cm Ejercicio 4.064) se acepta Ho. se distribuyen de modo gaussiano. 05 si la altura media es diferente de 174 cm. En este caso tenemos que Valor P=0.19) = 1. Se encontró una media de 40 kg y una varianza de 4 kg. x  t / 2   x  t / 2  20   n n 2   = = (39.06. Ejercicio 6.729) se rechaza Ho o sea la media es menor que 45kg b.05 Conclusión: Dado que Valor P es mayor que α entonces se acepta la Ho. paga un salario mensual de s/. Pero hay otra forma de tomar la decisión de aceptar la Ho y esto se hace con el Valor P. . Solución.18 s 2 n 20 x Datos n=20. 40. con un varianza de 4 días.1278) Valor T -2.09 .05 e.057 La interpretación es la misma que se hace en el caso manual ya visto anteriormente.09  2  20  40  2.94) Ejercicio 7. de lo contrario solo paga s/. Con objeto de estimar el peso de los niños de un jardín se tomó una muestra aleatoria al azar de 20 niños. d. =40.05/2=2.19) = 1.000 Intervalo de confianza al 95% (165. si la distribución de las ventas se considera normal.09 s s  .000 Cuadrado medio del error 2. s= 2 a. Intervalo de confianza a 95% t0. 40  2. con =0. y esta decisión es la misma que vimos anteriormente. Si Valor P ≤ α se rechaza la Ho de lo contrario se Acepta. Determine un intervalo de confianza al 95% para la media. con una confianza de 0. Pruebe la hipótesis de que la media es menor que 45kg.95.05.05.000 Desviación Estándar 10. Un empresario de una editorial. ¿Cobran s/1000 cada vendedor?. t  x  u 40  45    11 . Como (t=11.057 y α=0.5 por libro vendido. 174. un grupo de 10 vendedores vendió mas de 100 libros en 32 días.n Media 25 170.1000 si un vendedor vende mas 100 libros a lo mucho en 30 días. Ho: μ ≥ 45 (La media no es menor que 45 kg) H1: μ < 45 (La media es menor que 45 kg) t(0. Suponiendo que la muestra se distribuye normalmente.729 Conclusión.8722.00 Valor P 0.18)> (t(0. 833 Conclusión.8 100.0 100. Aplicando kolmogorov-Smirnov de minitab .797 Conclusión.797) no se rechaza Ho o sea la media de la ciudad es s/198. =200.24) = 2. Ejercicio 9. Primero determinamos si la distribución es normal.7 99.2 100.05.0 Verificar la Ho del fabricante con un coeficiente de riesgo del 5%. considerar la distribución normal.5 100. s=2 t(0.0 99.162 s 2 n 10 x Datos n=10.Solución x  u 32  30 t    3.7 100.1 100.0 98. con una confianza de 99%. s=4 Ho: μ = 198 (La media de la ciudad es s/198) H1: μ ≠ 198 (La media de la ciudad no es s/198) t(0.3 100. Solución t  x  u 200  198   2. Una muestra de 25 familias de una ciudad pagan en promedio por servicios s/ 200 mensual y una varianza de s/16.6 99.833) se rechaza Ho o sea no cobran los s/ 1000. Se toma una muestra de 15 paquetes siendo el resultado de la inspección el siguiente: 98.4 101. Como (t=2.24) = 2. Ejercicio 8.5 s 4 n 25 x Datos n=25. Un fabricante de mantequilla quiere comprobar si el peso promedio de cada paquete es de 100 g.5)< (t(0.2 99.9) = 1. Como (t=3.5 98.005.9) = 1. Ho: μ ≤ 30 (Cobran los s/1000) H1: μ > 30 (No cobran los s/1000) =30.05.005.162)> (t(0. Solución. Se trata de ver si esta muestra es consistente con la H o que la media en la ciudad por servicios es de s/ 198. podemos aplicar la t de estuden para probar nuestra hipótesis.025.828)> (t(0. El aumento de peso promedio de 18 vacas bajo una dieta alimenticia durante dos meses fue de 8 kg con una s=3kg.14) = 2. t  x  u 8  10    2.7442 Hipótesis Ho: μ = 100 (El paquete pesa en promedio 100 g) H1: μ ≠ 100 (El paquete no pesa en promedio 100 g) t(0.150 esto es mayor que 0.145) no se rechaza Ho o sea el paquete pesa en promedio 100 g.025.025.2 kg. Segundo t  x  u 99.5 kg. Se desea probar si es válido afirmar que esta ración aumenta el peso al menos en 10 kg. =99. con una desviación de 1.05. Como (t=1.Se tiene que p-value=0. .744 n 15 x Datos n=15.14) = 1.73.740 Conclusión.05. Un fabricante que elabora alimento balanceado. luego la distribución es normal.828 s 3 n 18 x Datos n=18. s=3 Ho: μ ≥ 10 (El peso aumenta al menos en 10 g) H1: μ < 10 (El peso no aumenta al menos en 10 g) De la tabla t se tiene t(0. s=0. si la distribución es normal ¿Que a comprobado el fabricante con un nivel de significación del 1% ? Solución. Ejercicio 9. Como (t=2. considerar distribución normal. Solución.145 Conclusión.14) = 2. durante los dos meses con un nivel de significación del 5%. desea comprobar que los pesos de los paquetes tienen un promedio 60kg. Ejercicio 10.14) = 1. =8.740) se rechaza Ho o sea el peso no aumenta al menos en 10 g.41)< (t(0.73  100    1. En una muestra de 9 paquetes tomados al azar se obtuvo una media de 61.41 s 0. t  x  u 61.2  62    1 .6 s 1.5 n 9 x Datos n=9, =61.2, s=1.5 Ho: μ = 60 (El peso promedio es 60 kg) H1: μ ≠ 60 (El peso promedio no es 60 kg) De la tabla t se tiene t(0,005;8) = 3.355 Conclusión. Como (t=1.6)< (t(0,005;8) = 3.355) no se rechaza Ho o sea el peso promedio es 60 kg. 9.2.4 EJERCICIOS PROPUESTOS Ejercicio 1. La corporación Tampico desea saber cuál es la máxima tensión de ruptura que soportan los cables de acero que fabrica. Un cliente importante está interesado en la compra de un número grande de cables y ha establecido que el punto de ruptura no debe ser menor que un tonelada. Tampico piensa que una tonelada es aproximadamente el punto de ruptura de los cables, pero decide probar la hipótesis de que la tensión de ruptura promedio es una tonelada. La muestra es de 10, =0.05, x= 0.96, s=0.15 Ejercicio 2. Se tiene las siguientes pruebas de hipótesis Ho: μ= 20 H1: μ≠ 20 Los datos de una muestra de 6 elementos son 16, 20, 18, 19, 20, 18 a. Calcule la media de la muestra b. Encuentre la desviación estándar de la muestra c. Con α=5%, cual es la regla de rechazo d. Calcule el valor del estadístico t e. ¿Cuál es su concusión? f. ¿Que puede decir acercad del valor p? Ejercicio 3. Se tiene la siguiente prueba estadística Ho: μ= 10 H1: μ< 10 Con una muestra de 15 datos se obtuvo una s=5, use α=5% determine el valor de t y su conclusión para cada uno de los siguientes resultados x a) x = 9 b) x = 11 c) x = 8.5 d) x = 9.5 e) = 12 Ejercicio 4. Un cardiólogo desea hallar un intervalo de confianza del 90% para el nivel colesterol promedio de todos los pacientes que presentan problemas cardíacos. Para esto asume que la distribución de los niveles de colesterol es normal con una desviación estándar  = 13 y usa la siguiente muestra al azar de niveles de colesterol de 20 pacientes con problemas cardíacos. 217 223 225 245 238 216 217 226 202 233 235 242 219 221 234 199 236 248 218 224 Ejercicio 6. El pH medio del agua que sale de una planta de tratamiento debe ser de 7.0. La autoridad sospecha que es posible que cierta planta no cumpla con la normativa. Se tomaron 15 muestras de agua de esa planta y se obtuvo un pH de 6.7, 7.1, 6.8, 6.9, 7.3, 7.5, 6.5, 6.6, 7.3, 7.1, 6.3, 6.8, 7.0, 7.1 y 6.8. Sabiendo que el pH varía según una distribución normal, ¿hay razón para dudar que se mantenga la especificación? Ejercicio 7. Dos secciones de un curso de estadística son sometidas a un mismo examen final. De las calificaciones obtenidas se extrae una muestra aleatoria de tamaño 9 en la grupo "A", y otra de tamaño 4 en el grupo "B". Grupo "A": 65, 68, 72, 75, 82, 85, 87, 91, 95 Grupo "B": 50, 59, 71, 80 a. Con un nivel de significación de 0.05 ¿podría decirse que los dos grupos tienen las mismas calificaciones promedio?. Suponga que provienen de poblaciones normales con varianzas iguales. Ejercicio 8. La experiencia en la investigación de demandas por accidentes en una institución aseguradora revela que en promedio cuesta $60 la realización de los trámites. Este costo se consideró exorbitante en comparación al de otras compañías aseguradoras y se instauraron medidas para reducir costos. A fin de evaluar el impacto de las medidas, se seleccionó una muestra de 16 demandas recientes. Se encontró un costo promedio de $57 y una desviación estándar de $10. Elabore una prueba de hipótesis que permita comprobar si los costos han disminuido, con un 99% de confianza. Ejercicio 9. Por registros pasados se sabe que la duración promedio de unas pilas eléctricas que se fabrican para ser utilizadas en un reloj digital es de 300 días. Hace poco tiempo, el proceso de fabricación fue modificado para darle mayor duración. Para comprobar la efectividad del proceso modificado, se probó una muestra de 20 pilas, y se encontró una duración promedio de 311 días y una desviación estándar de 12 días. A un nivel de significación de 0,05, ¿puede afirmarse que el nuevo proceso aumenta la duración de las pilas? Ejercicio 10. Un gerente de ventas de libros universitarios afirma que en promedio sus representantes de ventas realizan 40 visitas a profesores por semana. Varios de estos representantes piensan que realizan un número de visitas promedio superior a 40. Una muestra tomada al azar con 8 semanas reveló un promedio de 42 visitas semanales y una desviación estándar de 2 visitas. Utilice un nivel de confianza del 99% para aclarar esta cuestión. Ejercicio 11. Cuando la cantidad de semillas de soja que quedan en el suelo luego de pasar la cosechadora es igual o mayor a 80 semillas/m 2, la pérdida de producción, en qq/ha, es grande. Un productor decide probar el funcionamiento de su máquina y para ello luego de cosechar una parcela cuenta en 10 unidades de 1 m2 cuántas semillas quedan en el suelo. Los resultados fueron, en semillas/m2: 77 73 82 82 79 81 78 76 76 75 a) ¿Se puede concluir, trabajando con un nivel de significación del 10%, que la cosechadora está funcionando bien?, es decir, ¿está la perdida dentro de los límites admisibles? b) Construir un intervalo de confianza para μ apropiado para el problema. Ejercicio 12. Referido al problema 11. a) Si las normas técnicas indican que la desviación estándar del número de semillas caídas por m2 no debería ser superior a 5, ¿qué se debería concluir sobre la máquina trabajando con un nivel de significación α = 0.10? b) Construir un intervalo de confianza para σ2. Ejercicio 13. Los registros de una comercializadora de repuestos para vehículos revelaron que la duración promedio de un juego de bujías es de 44.000 kilómetros. Un fabricante de bujías, sin embargo, afirmó que su producto tiene una vida media superior a este valor. El propietario de una flotilla de camiones adquirió 18 bujías, como prueba. Encontró una duración promedio de 42.400 kilómetros y la desviación 1.500. Esta información muestral convenció al propietario. ¿Y a Ud.? Ejercicio 14. Una cadena de talleres para la afinación de motores de automóvil anuncia que su personal puede realizar el servicio completo (cambio de aceite, cambio del filtro de aceite, lavado y engrase de motor) en un promedio de 15 minutos. Sin embargo, la gerencia ha recibido quejas de los clientes en relación al tiempo de servicio. Para verificar la afirmación, la oficina muestreó a 21 automóviles, obteniendo una media de atención de 18 minutos y una desviación de 1 minuto. Utilice un nivel de significación de 0,05 para probar si es razonable la afirmación de la cadena de talleres. Ejercicio 15. Se instala una máquina para llenar botellas pequeñas con 9,0 gramos de medicamento. Se piensa que el peso medio es de menos de 9,0 gramos. Una muestra de llenado se da a continuación. Pruebe la afirmación con un 99% de confianza. 9,2 8,7 8,9 8,6 8,8 8,5 8,7 9,0 Ejercicio 16. Se desea estudiar el gasto semanal de fotocopias, de los estudiantes de la UNHEVAL. Para ello, se ha elegido una muestra aleatoria de 9 de estos estudiantes, resultando los valores siguientes para estos gastos: 100 150 90 70 75 105 200 120 80 Se supone que la variable aleatoria objeto de estudio sigue una distribución normal de media desconocida y de desviación típica igual a 12. Determina un intervalo de confianza del 95% para la media del gasto semanal en fotocopias por estudiante. Ejercicio 17. Se sabe que la desviación típica de las notas de cierto examen es 0,4. Para una muestra de 36 estudiantes se obtuvo una nota media de 5,6. ¿Sirven estos datos para confirmar la hipótesis de que la nota media del examen fue de 6, a un nivel de significación de 0,05? Ejercicio 18. Cuando fue contratado como mesonero de un restaurante, a un caballero se le dijo que obtendría S/.20 por día en propinas. Al cabo de 25 días de trabajo, el mesonero piensa que no obtuvo tanto en propinas. El restaurante le pidió cuentas de sus propinas durante el mes: había recibido S/. 450 en los 25 días. Así mismo, se determinó con los datos muestrales que la desviación estándar fue de S/. 3. por día. ¿Puede el mesonero sostener estadísticamente su opinión, al 95% de confianza? Ejercicio 19. Pruebe la hipótesis de que el contenido promedio de los envases de un lubricante en particular es de 10 litros si los contenidos de una muestra aleatoria de 10 envases son: 10.2, 9.7, 10.1, 10.3, 10.1, 9.8, 9.9, 10.4, 10.3 y 9.8 litros. Utilice un nivel de significancia de 0.01 y suponga que la distribución del contenido es normal. Ejercicio 20. Una muestra de 9 explotaciones agrícolas arrojó una media de 125 ha y un desvío de 25 ha. Testar si se puede suponer con bastante confiabilidad que el promedio verdadero de la población de explotaciones puede ser 135 ha. 3 PRUEBA DE HIPÓTESIS SOBRE LA DIFERENCIA ENTRE MEDIAS: MUESTRAS GRANDES 9.3.3. es decir. Estadístico de prueba a) Varianzas conocidas 12 y 22 b) Varianzas desconocidas 12 y 22 .2 SUPUESTOS 1.1 INTRODUCCIÓN Supóngase que se tiene dos poblaciones independientes con medias desconocidas μ1 y μ2. Las poblaciones son independientes 3. y x1 2 x2 2 varianzas σ1 y σ2 . Hipótesis Caso I Caso II Caso III Ho: μ1 – μ2 ≥ Δ Ho: μ1 – μ2 = Δ Ho: μ1 – μ2 ≤ Δ H 1 : μ1 – μ2 < Δ H 1 : μ1 – μ2 ≠ Δ H1 : μ1 – μ2 > Δ 9. Los tamaños de las muestras son n 1≥ 30 y n2 ≥30 4. Las observaciones de las muestras son aleatorias 2. Las poblaciones son normales o cumplen las condiciones del teorema del límite central.9. Sean y las medias de las muestras de dos poblaciones. Queremos observar si la diferencia entre las medias es significativa o no. El tamaño de cada una de estas muestras son n1 y n2 respectivamente. b) Se desconoce la varianza poblacional σ2. n2 : Tamaño de la muestras 2 x1 x2 : Media de la muestra 1 . 1.z x1  x 2  ( 1  u 2 ) 1  2  n1 n2 2 2 zt  x1  x 2  ( 1  u 2 ) 2 2 s1 s  2 n1 n2 Intervalo de Confianza. μ2 : Media de la población 2. para calcular el p-valor en la práctica se usa generalmente la distribución t porque en la mayoría de los paquetes estadísticos viene como una opción 2. s : desviación estándar de la muestra 2 Notas. Del 100(1-)% para la diferencia de medias poblacionales 1-2. Para muestras grandes es indistinto usar la distribución Z o distribución t de student. es de la forma: a) Se conoce la varianza poblacional σ2. 1 : Desviación estándar poblacional 1 . 2 : Desviación estándar poblacional 2 s1 : desviación estándar de la muestra 1. Si NO se conoce las desviaciones estándar de las poblaciones 1 y 2 se estima con s1 y Sha las .  2 2 2 2   x  x  Z 1   2      x  x  Z 1   2  1 2 1 2  /2  1 2  / 2 n1 n2 n1 n2     2 2 2 2 s1 s2 s1 s2   x  x Z      x x Z   1 2  / 2 n1 n2 1 2 1 2  / 2 n1 n2    n x Donde: n1: Tamaño de la muestras 1. : Media de la muestra 2 μ1 : Media de la población 2. Generalmente se tiene que μ1 – μ2 = 0 9.025=-1.025 =1.38 Zona rechazo Zona Ho de aceptación Zona rechazo Ho Ho Z0. a un nivel de significación del 0. n2 = 56. Las dos poblaciones siguen una distribución Normal N(μ 1. y para el criterio de aceptación tenemos en la figura: Conclusión. utilizaremos el siguiente estadístico: z x1  x 2  ( 1  u 2 ) 2 2 s1 s  2 n1 n2 Estadístico  74  78  (0) 225 169  42 56  1.3 EJERCICIOS RESUELTOS Ejercicio 1.μ 2 = 0. μ 1 = μ 2 (no existe diferencia entre las poblaciones) H1: μ 1 . luego aceptamos Ho. es decir no existe diferencia entre las poblaciones Solución en MINITAB Ejecutamos las opciones Stat/Basic statistics/2 Sample t.96 Z= – 1.025. las varianzas S12 = 225 . ok . X2 = 78 . pero desconocemos el valor de su desviación.05 como es de dos colas α/2 = 0.05.μ 2 ≠ 0. que son los siguientes: Las medias X1 = 74 . Hipótesis: Ho: μ 1 .. Se pide contrastar estadísticamente si existe diferencia entre las dos poblaciones. es decir. luego completamos los datos como se ve en la figura y clickeamos ok.. μ 1 ≠ μ 2 (si existe diferencia entre las poblaciones) Ya que el tamaño de las muestras es elevado.3.38) queda en el área de aceptación de Ho. S22 = 169. Como (Z = – 1. es decir. Se conocen los datos de dos muestras de dos poblaciones. las muestras n 1 = 42 . σ22) Solución. sólo conocemos el valor de la desviación típica de las muestras entonces estimamos las desviaciones poblacionales con las de las muestras. Sabemos que las distribuciones de las dos poblaciones son Normales.desviaciones estándar de las muestras 3.σ12) y N(μ2.38 El nivel de significación nos dice el enunciado que es de α = 0.96 Z0. 171) > (α=0.Resultados Two-Sample T-Test and CI Sample 1 2 N 42 56 Mean 74.mu (2) Estimate for difference: -4.0 13. 1. Con un nivel de significancia de 0.171 DF = 81 Conclusión. Como (p = 0.0 SE Mean 2.0 StDev 15. Se realizó un estudio para comparar los años promedio de servicio de quienes se retiraron en 2005 con los que se retiraron el año anterior en un hospital.38 P-Value = 0. Ejercicio 2.75807.75807) T-Test of difference = 0 (vs not =): T-Value = -1.00000 95% CI for difference: (-9.3 1.01 .05) luego no se rechaza Ho.0 78.7 Difference = mu (1) . 3263) se rechaza Ho.0 40.8 Concusión: Como (Z= -6.4  (0) 2.6 2  40 45  6.8) < (Z0. HIPÓTESIS Ho: μ 2 .9 2 3.40 25.0 Solución.μ 1 ≥ 0.9 Tamaño de la muestra 45. (los trabajadores del 2005 trabajan igual o mas que los del 2004) H1: μ 2 .μ 1 < 0. (los trabajadores del 2005 trabajan menos que los del 2004) z x 2  x1  (  2  u1 ) 2 2 s2 s  1 n2 n1  25.6  30.60 Desviación estándar muestra 3. luego los trabajadores del 2005 trabajan menos que los de 2004 .6 2.01= 2.¿podemos concluir que los trabajadores que se retiraron el 2005 trabajaron menos que los del 2004. según la siguiente muestra? Año 2004 2005 Media de la muestra 30. 00)<(α=0.9.60 2.90 0.80000 99% upper bound for difference: -3.12520 T-Test of difference = 0 (vs <): T-Value = -6. eficaz el tratamiento? Solución. es decir hay suficiente evidencia para suponer que los trabajadores del 2005 trabajan menos que los del 2004. La variable medida es la disminución de la presión sistólica y se obtiene: grupo placebo x ha n = 35.01).60 0. Se trata de un contraste sobre diferencias de medias H0: μ1 – μ2 ≤ 0 (no varia la presión) H1: μ1 – μ2 > 0 (la presión disminuye) x 15.46 2 45 30. 3.Utilizando MINITAB Two-Sample T-Test and CI Sample N Mean StDev SE Mean 1 40 25.80 P-Value = 0.7 mm de Hg. En un ensayo clínico para evaluar un hipotensor se compara un grupo placebo con el grupo tratado. y Sha = 12. ¿Es .8.000 DF = 82 Conclusión: como (p-value=0.1 mm de Hg.40 3. y S = 33. grupo tratado n = 40. se rechaza Ho.54 Difference = mu (1) . Ejercicio 3.mu (2) Estimate for difference: -4. 8  35 40  10.82 3.645).05= – 1.7  15. los resultados serán iguales a la t de Students.58 SE Mean 0. Utilizando MINITAB : Two-Sample T-Test and CI Sample 1 2 N 35 40 Mean 3. z x1  x 2  ( 1  u 2 ) 2 2 s1 s  2 n1 n2  3. se aproxima a Z utilizamos la distribución Z.70 15.22 Luego tenemos que (Z = – 10.57 Difference = mu (1) .Como no conocemos σ1 ni σ2 utilizamos la s1 y Sha y la distribución t.1 33. significa que la presión con el nuevo tratamiento no ha disminuido.22) < (Z0.9 12.mu (2) .98 0.10 StDev 5. pero como la distribución t para muestras n>30. entonces se ACEPTA la Ho. En la actualidad las calificaciones de eficiencia de 50 trabajadores de una empresa tienen un valor promedio de 14. Sin embargo. Un producto químico especialmente diseñado para añadir peso al grano desea determinar si es o no eficaz. Ejercicio 6. existen diferencias entre las mismas.6 100 Compañía 2 1045 57.3.1 64 Ejercicio 3. para ello se tomaron 30 sabanas del mismo tamaño y color y se efectuó la prueba. 9. Utilizando un nivel de significación de 0. o si por el contrario. Un fabricante de detergente afirma que su producto rinde más que los de la competencia.04 P-Value = 1.5 puntos y una desviación estándar de 1. Una muestra de los últimos 36 meses reveló que ahora se surte 5.2 onzas.2998 T-Test of difference = 0 (vs >): T-Value = -10. Una muestra de 45 atenciones de la cadena McPollo arrojó un tiempo promedio de atención de 4 minutos con desviación de 0. ¿Ha cambiado la tasa de surtido? Utilice un nivel de confiabilidad del 98%. ¿es posible concluir que el producto es eficaz? Ejercicio 4. La presión no varía. encontrándose que para un lavado perfecto se requirió de 800g de detergente con 15 de desviación estándar en un tiempo dado. Ejercicio 2.10. Con base a un muestreo.8 minutos ¿Tiene McPato razón? Utilice una prueba estadística con un nivel de significación de 0. Dos compañías que fabrican bloques de concreto desean comparar la compresión promedio de sus bloques.5 veces. en fechas recientes se evaluó a 64 trabajadores.02.2 onzas.05) luego se acepta Ho. con una desviación de 1 onza.4000 95% lower bound for difference: -13. Dentro de un nivel de confianza del 95%. con una desviación de 1. con una desviación de 1. 30 sabanas del mismo tamaño y color se probaron para el detergente de la competencia arrojando para un lavado perfecto en el mismo tiempo un promedio de 860g con desviación estándar de 10. ¿es posible concluir que hay diferencias entre la compresión promedio de los bloques de ambas compañías? Compresión promedio(psi) Desviación estándar (psi) Tamaño Muestra Compañía 1 1070 63.4 EJERCICIOS PROPUESTOS Ejercicio 1. una muestra de 42 atenciones demoró en promedio de 3 minutos con desviación típica de 1 minuto. El interés es determinar si las dos compañías tienen compresiones promedio iguales. Las existencias de un medicamento se han surtido siempre en una farmacia un promedio de 6.4 veces al año. Una muestra de 100 mazorcas de maíz no tratado con el producto tuvo un peso promedio de 15.000 DF = 54 Conclusión. Utilizando una prueba de hipótesis con un 95% de confiabilidad.2 veces al año. Se tomaron dos grupos: el primer grupo fue formado con grano al cual no se le aplicó el producto y el segundo grupo fue de grano al cual si se le aplicó el producto.6 puntos. De aquí tenemos que (p-valor =1) > (α=0. Una muestra de 400 mazorcas de maíz tratado con el producto tuvo un peso promedio de 16 onzas. .5 puntos y una varianza de 9. ¿Tiene razón el fabricante? Utilice un nivel de confianza del 99%. que obtuvieron una puntuación media de 16. ¿puede decirse que la puntuación actuales de los trabajadores de la empresa es menor que en fechas recientes?. Ejercicio 5. pudo determinarse la información que sigue.Estimate for difference: -11. La cadena de McPato situadas en la ciudad de Piura afirma que su servicio es mas rápido que cualquier otra cadena. Se sospecha que esta tasa ha cambiado en los últimos meses. En Huánuco una muestra de 30 cabinas de Internet dijeron que ganaban en promedio diario S/250 con una desviación típica de 2. con una desviación de S/.2. 100 Empleados de una casa comercial matriz situado en la ciudad de lima ganan por comisiones en promedio s/500 mensuales con desviación estándar de 12.1 años. 370.Ejercicio 7. a un nivel de confianza de 90%. En la zona norte se visitaron 200 domicilios de las mismas características seleccionados al azar y el 38% de ellos estaba dado de alta en Internet. Para medir cambios experimentados desde el año anterior. ¿Estaría justificado afirmar que en la zona norte hay más gente conectada a Internet que en la zona sur? ¿Cuál debería haber sido el tamaño de la muestra para poder detectar una diferencia de al menos 5 puntos porcentuales el 90% de las veces. ¿A que conclusión deberá llegar usted? Ejercicio 13. Una muestra de 35 departamentos en la Avenida principal de provincias proporcionó un alquiler mensual de S/. Otra muestra B de 36 observaciones arrojó una media igual a 12 y una varianza de 2. ¿La media de la muestra A es menor que la muestra B? Ejercicio 8. Una muestra de 40 departamentos en Lima demostró un valor promedio de S/. con una desviación de 2.2. Una muestra de 55 casas en El Naranjal señaló un tiempo promedio de 18. y se compara con una muestra de 50 casas en el mes de enero del 2005. con un nivel de significación de 0.05? Ejercicio 12. con una desviación de 1. Se quiere saber si el aumento del precio del petróleo genera incrementos en el uso de electricidad. 30..12. Con un nivel de significación del 5%.9 años. Ejercicio 9. con una desviación de 2. Se incorporó un nuevo método de aparado y sobre una muestra de 32 zapatos requiere en promedio 9 minutos con varianza de 4. Este número descendía al 29% en la zona sur donde se visitaron 240 hogares. La gerencia de una fábrica está considerando un nuevo método de aparado en la fabricación de zapatos. con una desviación de S/. 30 empleados en Huancayo de una sucursal ganan por comisiones s/450 mensual con desviación típica de 10. determinar si en las cabinas de Internet de Amariles se gana más que en las de Huánuco. determinar si el nuevo método es mas eficaz a un nivel de confianza de 99%. Una empresa de bienes raíces está preparando un folleto que cree que puede ser de interés para compradores de apartamentos en Las Palmeras y El Naranjal. Un elemento de interés es el tiempo que el propietario que vende ha ocupado el inmueble. los resultados muestrales son: x1  1. 680. se elige una muestra al azar de 40 casas en el mes de enero de 2004.05 probar la hipótesis de que la sucursal paga menos que la casa matriz.8.5 minutos de aparado y una desviación típica de 3 para tres docena de zapatos. Se realiza una encuesta en dos zonas distintas de un país para conocer el grado de implantación de Internet en los hogares. Una muestra de 40 apartamentos en las Palmeras indicó un tiempo promedio de permanencia de 17.925kwh s 2  305 kwh n 2  50(2005) Realice una prueba con un nivel de significancia de 0..10 para verificar si el promedio de consumo de electricidad por casa ha cambiado durante el mes de enero. Ejercicio 14.6 años. El método actual requiere en promedio de 12. ¿se puede concluir que los residentes de Las palmeras tenían en propiedad sus casas por un período más corto? Ejercicio 10. ¿Se puede concluir que existen diferencias entre los alquileres? Ejercicio 11. . En una muestra aleatoria de 35 cabinas de Internet en la ciudad de amarilis ganan en promedio diario S/200 con una varianza de 4. Un estudio se realiza para comparar el alquiler mensual de un departamento de una habitación en la Avenida principal de provincias contra el costo en Lima.645kwh s1  298 kwh n1  40(2004) x 2  1. con un nivel de significación de 0.3 años. Una muestra A de 49 observaciones muestrales reveló una media de 7. Ejercicio 15. a un nivel de confianza de 95% se puede afirmar que el 2007 se incremento las exportaciones de mango. . Las exportaciones de mangos del 2006 de 30 empresas fueron en promedio de 800 toneladas métricas. con una varianza de 200 toneladas métricas al cuadrado. el 2007 estas mismas empresas exportaron 1000 toneladas con una varianza de 150. 4 PRUEBA DE HIPÓTESIS SOBRE LA DIFERENCIA ENTRE MEDIAS: MUESTRAS PEQUEÑAS 9. Caso 1.2 SUPOSICIONES 1. Al menos una muestra es pequeña n < 30 Prueba Estadística.4.9.4. Del 100(1-)% para la diferencia de medias poblacionales 1-2.1 INTRODUCCIÓN Estas pruebas se utilizan cuando el muestreo destruye a los elementos. Las observaciones de las dos muestras son independientes 2. Se conocen las desviaciones estándar de las poblaciones σ1 y σ2. x1  x 2   o z 1  2  n1 n2 2 2 Intervalo de Confianza. es de la forma:   12  12  12  12   x  x z       x  x  z  2  /2 1 2 1 2  /2  1 n n n n1  1 1 1  . Se quiere probar la hipótesis sobre la diferencia de medias bajo el supuesto que Ho es cierto es decir: Hipótesis Caso I Ho: μ1 – μ2 ≥ Δo Caso II Ho : μ1o – μ2 = Δo H1 : μ1 – μ2 ≠ Δo H1 : μ1 – μ2 < Δo Caso III Ho: μo – μ2 ≤ Δo H1: μ1 – μ2 > Δo 9. Las dos poblaciones son aproximadamente normales 3... las medias de dos poblaciones normales o aproximadamente normal. cuando resulta muy costoso o cuando solo se puede obtener unos cuantos valores históricos. Sea u1 y u2. en función de s1 y S2. Del 100(1-)% para la diferencia de medias poblacionales 1-2. se determina s la desviación estándar combinada. Se desconoce 1 y 2 pero desiguales 1≠2 . para estimar 1 y 2 respectivamente en este caso el método aproximado es la distribución t. es de la forma:   x1  x 2  t / 2. Se desconoce 1 y 2 pero son iguales 1=2=. n1 n 2  2 s  n1 n1 n1 n1  n x Prueba t con n1+n2 – 2 grados de libertad Donde s es un estimado conjunto de σ (desviación estándar común para ambas poblaciones (pooled variance)) 2 (n1  1) s1  ( n2  1) s 2 s n1  n2  2 2 Caso 3.n1 n 2  2 s   1 1 1 1    1   2  x1  x 2  t / 2. se utiliza s1 y Sha. .n x Caso 2. t x1  x 2   o 2 2 s1 s  2 n1 n2 Los grados de libertad (gl) se determina con la fórmula siguiente. t x1  x 2   o s 1 1  n1 n2 Intervalo de Confianza. 10 . El lector cree tener suficiente razones para considerar las desviaciones estándar de población iguales. 2 : Desviación estándar poblacional 2 Sha : desviación estándar de la muestra 2 s: desviación estándar combinada. gl  1 n1 n1 n1 n1   n x Donde: n1: Tamaño de la muestras 1. las poblaciones son normales. La calificación media para el grupo A (10 pacientes) es 80 con desviación estándar 18. Del 100(1-)% para la diferencia de medias poblacionales 1-2. n2 : Tamaño de la muestras 2 x1 x2 : Media de la muestra 1. gl  2  / 2 . es de la forma:  s12 s12 s12 s12   x  x t   1   2  x1  x 2  t / 2 . 1 : Desviación estándar poblacional 1 s1 : desviación estándar de la muestra 1. y la correspondiente al grupo B (15 pacientes) es 70 con desviación estándar 22.3 EJERCICIOS RESUELTOS Ejercicio 1. Como psicólogo de un hospital para enfermos mentales el lector obtiene calificaciones para una prueba visual motora para cada uno de dos grupos de pacientes. Solución Datos nA = 10 Hipótesis x A  80 nB = 15 x B  70 sA = 18sB = 22  = 0.gl   s12 s 22      n1 n 2  1  s    n1  1  n1  2 1 2  2 1  s    n 2  1  n 2  2 2 2 2 Intervalo de Confianza.4. μ1 : Media de la población 2. gl : grados de libertad Nota: en caso gl no sea entero se aproxima al menor entero 9. : Media de la muestra 2 μ2 : Media de la población 2. ¿Difieren significativamente las calificaciones con nivel de significación 10%?. 1. 24.0000 90% CI for difference: (-4.3630.1 .7 Difference = mu (1) .5278 10  15  2 s= Zona rechazo Zona Ho de aceptación Zona rechazo Ho Ho T(0.0 StDev 18.3630) T-Test of difference = 0 (vs not =): T-Value = 1.0 22. 23) = 1. luego completamos los datosu1-u2 como se ve en la figura y clickeamos ok.714 Decisión: como se observa en la figura el t esta dentro del área de aceptación de Ho.1.μB = 0 (las calificaciones no difieren) H1: μA .245 DF = 23 .1 .7 5.23)=-1. luego se acepta Ho.0 SE Mean 5. 10+15-2) = t (0. es decir que las calificaciones no difieren Utilizando MINITAB Ejecutamos las opciones Stat/Basic statistics/2 Sample t..23) =1.. n1+n2-2) = t (0.μB ≠ 0 (las calificaciones si difieren) t x A  xB  1 1 s  n A nB 80  70 1 1 20.0 70. ok H1 x1 n1 n2 1=2 s1 x2 s2 Nota: Note que Assume equal viariances está activado esto se hace cuando las varianzas de las poblaciones son iguales Two-Sample T-Test and CI Sample 1 2 N 10 15 Mean 80.5278  10 15 2 2 (n A  1) s A  (n B  1) s B  n A  nB  2  1.mu (2) Estimate for difference: 10..714 t= 1.193 (10  1)18 2  (15  1)22 2  20.19 P-Value = 0.193 t(0.De tablas t para dos colas tenemos que: t (α .714 Ho: μA . 6928 1 6 2  1 (n A  1) s A  (n B  1) s B  n A  nB  2  0.1  5  8.5 0.5 7.10)=1.1 5 8. Solución. luego Con el proceso B no se disminuye el tiempo de ejecución T(0.5  7  8.5278 Conclusión: Como (p-value = 0.3  7. 25  2.10) = 1.5 6  2.05.1  4  8  6.25 2  (5)2. Ejercicio 2. medidos en horas: A 2.13 t(0.5)  (7 )  (8.6  5 )  8(5.1  4  8  6.68 esta en la zona de aceptación de la Ho.68 Conclusión: En la gráfica vemos que t=0.37 ) 6 1 2 2 2 2 2 2 2 ( 2.5)  (7.6 5 Admitiéndose normalidad y con un nivel de confianza del 95% ¿qué conclusión puede derivarse de estos datos?.5 7 8.1)  (5)  (8.1  6.3 7.6  5  5.Both use Pooled StDev = 20.6928 662 Zona de rechazo Ho Zona de Aceptación Ho t=0.1 4 8 6. Ho: μA – μB ≤ 0 (Con el proceso B no se disminuye el tiempo de ejecución) H1: μA – μB > 0 (Con el proceso B se disminuye el tiempo de ejecución) xA  sA  sB  2. se ejecutaron 6 tareas con ambos procesos obteniéndose los siguientes tiempos. Para contrastar si mediante el proceso B se disminuye el tiempo de ejecución de ciertos trabajos respecto del A.1) luego se acepta Ho.05.1) )  6(6.68 6 (5)2.37  5.812 una cola x A  xB t s 1 nA  1 nB 2  6.245) > (α = 0.13 2  0.3  7.37 6 xB  2 2 2 2 2 2 2 (( 2.5  7. Suponer que ambos procesos tienen la misma variabilidad.5) 6 1 2.1 B 2.812 . 427686 T-Test of difference = 0 (vs >): T-Value = 0.mu (XB) Estimate for difference: 0. Se quiere saber el trabajo afecta el rendimiento académico de los estudiantes de una especialidad. Ejercicio 3.50 2. supóngase que las poblaciones son normales.13 0. Como (p-value=0. El grupo 1 es el de estudiantes que trabajan y el grupo 2 es el de estudiantes que no trabajan. los datos obtenidos en la investigación son los siguientes.En MINITAB Primero ingresamos los datos en la columna C1 (XA) y C2 (XB) como en la figura Y ejecutamos las opciones Stat/Basic statistics/2 Sample t.. para determinar si son iguales aplicamos la prueba de la varianza. luego completamos los datos como se ve en la figura y clickeamos ok. XB Two-sample T for XA vs XB N Mean StDev SE Mean XA 6 6. G1 G2 5 10 12 10 8 16 11 17 12 15 13 16 8 14 11 16 Con un nivel de significancia de 0.. Solución 1: media de estudiantes que trabajan 2: media de estudiantes que no trabajan Primero: No se conocen las 1 y 2 además no sabemos si son o no iguales.1926 P-Value = 0.25 0. Two-Sample T-Test and CI: XA.255)>(α =0.37 2.68 Both use Pooled StDev = 2.255 DF = 10 Conclusión. puede afirmarse que el trabajo disminuye el rendimiento académico.92 XB 6 5.01. para ello se evalúa a dos grupos independientes de estudiantes.866667 95% lower bound for difference: -1. ok Aquí los resultados.05) se acepta Ho. .87 Difference = mu (XA) . 09028 g2 8 1.73 0.10 P-Value = 0.97.00.25000 99% upper bound for difference: 7. p-value = 1.2 < 0 (el rendimiento académico disminuye) Aplicando Minitab tenemos.72554 8.value=1)>0.mu (g1) Estimate for difference: 4.55800 2.20614 F-Test (normal distribution) Test statistic = 0.7451 .98 g1 8 10. g2 99% Bonferroni confidence intervals for standard deviations N Lower StDev Upper g1 8 1.996 DF = 14 Both use Pooled StDev = 2.76 0.971 Levene's Test (any continuous distribution) Test statistic = 0. p-value = 0. g1 Two-sample T for g2 vs g1 N Mean StDev SE Mean g2 8 14.76 Segundo.96 Difference = mu (g2) .76457 8.000 Como (p.01 se acepta la Ho. Como son iguales las 1 y 2 aplicamos la prueba de student con varianza combinada. Two-Sample T-Test and CI: g2.53600 2.85228 T-Test of difference = 0 (vs <): T-Value = 3. t x1  x 2 1 1 s  n n2 1 Hipótesis Ho: 1 . Además S1=2.Ho: 1 / 2 = 1 (la varianzas son iguales) H1: 1 / 2 ≠ 1(la varianzas no son iguales) Aplicamos Minitab Los resultados fueron Test for Equal Variances: g1.2 = 0 (el rendimiento académico no disminuye) H1: 1 .73 Sha=2.00 2. o sea que las varianzas de las poblaciones son iguales.25 2. 89447 2 5 0.05 se rechaza Ho. el río A esta ubicado en una zona industrial y el río B esta ubicada en una zona rural.01) se rechaza Ho.05 se desea saber si el río A esta más contaminado que B.047 Como (p-valor=0.004)< (=0.836660 2.468406 0.Nota: aquí hemos puesto g2 primero y luego g1. o sea que el trabajo diminuye el rendimiento académico de los estudiantes. Se determinó la contaminación de dos ríos A y B de una ciudad analizando el PH de 100 ml de agua. Ejercicio 4. p-value = 0. Solución Primero.91531 F-Test (normal distribution) Test statistic = 10.00. porque Sha>s1 Luego como (p-value=0. Utilizamos la comparación de varianzas Test for Equal Variances 95% Bonferroni confidence intervals for standard deviations Sample N Lower StDev Upper 1 5 0. Las distribuciones son normales.264575 0.047) < 0. Se dice que el agua no esta contaminada si su PH esta cercano a 7 Los datos encontrados fueron Río n x Sha A B 5 5 5 7 0. t x1  x 2  ( 1  u 2 ) 2 2 s1 s  2 n1 n2 Como  1≠  2 utilizamos la distribución Ho: 1 = 2 (El río A no esta más contaminado que el río B) H1: 1 < 2 (El río A esta más contaminado que el río B) Según minitab tenemos . Debemos determinar si las varianzas son iguales. pero no se sabe si las varianzas son iguales.7 0. luego las distribuciones No son iguales.148123 0.07 Con un =0. es decir el río A está mas contaminado que el río B. en particular si las muestras de las poblaciones no son iguales.031 Difference = mu (1) . los resultados obtenidos fueron: N: X = 28 kg S² = 9 NP: X = 21 kg S² = 7 Interesa conocer si existen diferencias significativas entre los rendimientos de las plantas tratadas con los dos tipos de fertilizante. Dieciocho plantas de una misma variedad de naranjos fueron tratadas con fertilizantes. A nueve de ellas se les aplicó una cierta dosis de nitrógeno (N) y al resto una de nitrógeno y fósforo (NP). Si el supuesto no fuera válido debería verificarse primeramente la homogeneidad de variancia a través del test F.05 luego se rechaza Ho. (α = 0.000 0. se calcula la variancia amalgmada. por planta.mu (2) Estimate for difference: -2.Two-Sample T-Test and CI Sample N Mean StDev SE Mean 1 5 5.828427 992 . Ejercicio 5.32930 T-Test of difference = 0 (vs <): T-Value = -6.002 < 0.31 2 5 7. s Donde ( n1  1) s12  ( n2  1) s 22  n1  n 2  2 (9  1)9  (9  1)7  2.700 0. Suponga varianzas iguales Solución: H0 : µ N = µ NP H1 : µ N µ NP Dado que las variancias poblacionales son iguales.0700 0. Se midió el rendimiento en Kg.36 P-Value = 0.002 DF = 4 Como p-value=0.00000 95% upper bound for difference: -1. de las cuales S² N y S²NP son estimaciones.01).0000 0. 25 El valor tabulado de t.05 se rechaza Ho. Como 1 ≠ 2 utilizamos la distribución t x1  x2  ( 1   2 ) 2 2 S1 S  2 n1 n2 gl   S1 2 S 2 2     n  n  1 2   2 1  S1  n1  1  n1  2 2 1  S 2  n 2  1  n2 2     2 .293052 0.4 0. Los resultados fueron: Individuos normales n1=20 x1  1.150964 0. para 16 grados de libertad y nivel de significación del 1% es igual a ± 2.2 La cirrosis de hígado. Se eligieron dos muestras aleatorias e independientes de individuos. Debemos determinar si las varianzas son iguales.619718 2 25 0. siendo superior el promedio por planta de naranjo. Conclusión existen diferencias estadísticamente significativas entre los tratamientos.2 0.8 S1=0. p-value = 0.66 S2=0. Utilizamos la comparación de varianzas Test for Equal Variances 95% Bonferroni confidence intervals for standard deviations Sample N Lower StDev Upper 1 20 0. Se quiso probar si la cirrosis de hígado hacia variar el índice de actividad de la colinesterasa en suero.002 Como (p-valor=0. ¿hace variar el índice de la colinesterasa en suero? Solución: Primero. luego las distribuciones No son iguales.828427 9  5. se rechaza H 0 .002) < 0. Como el valor de la estadística calculada supera al valor tabulado.00. Ejercicio 6.4 Individuos cirroticos n2=25 x 2  0. de aquellas que reciben el tratamiento NP.t ( X N  X NP )  (  N   NP ) 1 1 s  n N n NP  (28  21) 2 2.292749 F-Test (Normal Distribution) Test statistic = 4.921. Hipótesis: 1   2 H0: 1   2 H1: (No varía) (Varía) Según Minitab tenemos Two-Sample T-Test and CI Sample N Mean StDev SE Mean 1 20 1.22499 6. es decir el índice de la colinesterasa en suero no hace variar la cirrosis de hígado.400 0.27061 7.mu (2) Estimate for difference: 1.64.9386. Utilizamos la comparación de varianzas Test for Equal Variances 99% Bonferroni confidence intervals for standard deviations Sample N Lower StDev Upper 1 19 5.279) < 0.3540 F-Test (Normal Distribution) Test statistic = 1.000 < 0.000 DF = 26 Como P-Value = 0.05 se rechaza Ho. luego las distribuciones No son iguales.200 0.1 10. Muchos autores afirman que los pacientes con depresión tienen una función cortical por debajo de lo normal debido a un riego sanguíneo cerebral por debajo de lo normal. p-value = 0.279 Como (p-valor=0.8 S2=6. 1.9551 2 22 4.660 0.3414) T-Test of difference = 0 (vs not =): T-Value = 11.1400 95% CI for difference: (0.089 2 25 0. unos con depresión y otros normales.8 Normales n2=22 x 2  53.05 luego se rechaza Ho.1 ¿Hay evidencia significativa a favor de la afirmación de los autores? Solución: Primero.040 Difference = mu (1) . se les midió un índice que indica el flujo sanguíneo en la materia gris (dado en mg/(100g/min)) obteniéndose: Depresivo n1=19 x1  47 S1=7.64 P-Value = 0.8 13. Como 1 ≠ 2 utilizamos la distribución .800 0. A dos muestras de individuos. Ejercicio 7. Debemos determinar si las varianzas son iguales. 998 > 0.07 P-Value = 0.3 Difference = mu (1) . Si las poblaciones se consideran normales de varianzas desconocidas diferentes ¿tiene razón el fabricante? Utilice un nivel de confianza del 99%.10 1.t x1  x2  ( 1   2 ) 2 gl  2 S1 S  2 n1 n2  S1 2 S 2 2     n  n  1 2   2 1  S1  n1  1  n1  2 2 1  S 2  n 2  1  n2 2  2    Hipótesis: 1   2 H0: 1   2 H1: (función cortical Normal) (Función cortical por debajo de lo normal) Según Minitab tenemos Two-Sample T-Test and CI Sample N Mean StDev SE Mean 1 19 47.05 luego se acepta Ho. Solución: Fabricante n1 = 5 x1  60000 V1 = 100 Competencia n2 = 6 x2  58000 V2 = 90 Utilizamos la distribución t x1  x2  ( 1   2 ) 2 2 S1 S  2 n1 n2 gl  1  n1  1   S1 2 S 2 2     n  n  2   1 2 2  S 2 S1    1  2 n1  n 2  1  n2 2     2 .998 DF = 33 Como P-Value = 0. Un fabricante de llantas para bicicleta afirma que sus llantas duran más que los de la competencia.80 95% lower bound for difference: -10.000 kilómetros y una varianza de 100.54 T-Test of difference = 0 (vs >): T-Value = -3.80 6.mu (2) Estimate for difference: -6. Ejercicio 8.00 7. para ello se tomaron 5 llantas y la duración promedio fue de 60. 6 llantas de la competencia arrojo un tiempo promedio de duración de 58000 kilómetros y una varianza de 90.8 2 22 53.80 1. es decir no hay evidencia significativa a favor de la afirmación de los autores. 00 9.9 Difference = mu (1) . Inc.4.Hipótesis: 1   2 H0: H1: 1   2 (No duran más) (Duran más) Según Minitab tenemos Two-Sample T-Test and CI Sample N Mean StDev SE Mean 1 5 60000.0 10. Machos 44 48 36 32 51 45 54 56 Hembras 32 40 44 44 34 30 26 Ejercicio 2.000 < 0.01 luego se rechaza Ho. pero el profesor quiere una conclusión rápida.000 DF = 8 Como P-Value = 0. Así que observó en su planilla las siguientes calificaciones: Sección “A”: 12 05 13 16 15 10 . fundamentado en una muestra pequeña.4 EJERCICIOS PROPUESTOS Ejercicio 1.45 2. Ejercicio 3. Los pesos netos de las botellas de una muestra que llenó una máquina fabricada por Blue y los pesos netos en una muestra de botellas llenadas por una máquina similar que manufactura Red.86 T-Test of difference = 0 (vs >): T-Value = 338. Se mide la longitud de animales machos y hembras y se desea conocer si las longitudes son diferentes. Una muestra de las calificaciones que presentaron hombres y mujeres en un examen de Estadística se sintetiza a continuación: Hombres Mujeres Media muestral 11.49 3. 9. es decir que si tiene razón el fabricante que sus llantas duran más. o por el contrario.00 99% lower bound for difference: 1982. Ambas secciones tuvieron 60 estudiantes.05 que el peso medio de las botellas que llena la máquina que fabrica Red es mayor.01 P-Value = 0.50 Desviación estándar 3. existe alguna diferencia entre ellas? Responda a esta pregunta con una prueba estadística de hipótesis que tenga un nivel de confianza del 99%.33 10. Ejercicio 4.mu (2) Estimate for difference: 2000.5 2 6 58000. Un profesor está comparando las notas de dos secciones de Estadística.0 4.35 Tamaño de la muestra 6 8 ¿Son las calificaciones promedios iguales. fueron los siguientes: Blue: Red: 5 8 8 10 7 7 6 11 9 9 7 12 14 9 Pruebe al nivel de significación de 0. 3 5. dentro de un nivel de confianza del 95%.8 5 5.2 1. pero como los niveles de interpretación son muy variados en la plantación.0 27. comente sobre las ventajas y desventajas de uno y otro procedimiento.3 2.4 24.5 23.5 3.2 25.1 2 0.1 24. Los resultados se presentan a continuación: Salmón Atún Contenido de proteínas en porcentaje 22. c) Compare los resultados obtenidos.01 Suponga que las varianzas de las dos poblaciones son iguales.7 12.1 5. b) Considerar ahora a las muestras como independientes (no apareadas) y aplicar la prueba t para esta situación.0 6 4.4 28.9 4 1.9 Esta información es suficiente para decir que ambos pescados enlatados tienen el mismo contenido de proteína? Ejercicio 7.4 1. Al medir el diámetro transversal del corazón de los adultos del sexo masculino y femenino se obtuvieron los siguientes resultados: Grupo Tamaño de muestra Media muestral (cm) S en cm Hombres 12 13.4 7 10.4 25. Para ello se tomaron dos muestras de 7 unidades para cada uno de los dos productos y se realizó la determinación del contenido proteico en lo mismo.9 8 1. Se desea comparar la actividad motora espontánea de un grupo de 25 ratas control y otro de 36 ratas desnutridas.05 Mujeres 9 11 1.1 a) Analizar la información planteando una prueba t de muestras apareadas. que el promedio de notas de las dos secciones es igual? Ejercicio 5.7 26. Los datos obtenidos fueron los siguientes: . Se comparan dos insecticidas A y B.3 24.5 3 3. Se midió el número de veces que pasaban delante de una célula fotoeléctrica durante 24 horas. Para dos tipos de combustibles de automóvil se tomó el rendimiento por galón de cada uno y se presenta a continuación: Combustible A: 45 67 54 41 38 59 48 31 59 31 50 Combustible B: 79 82 69 84 76 77 81 65 73 70 69 Con base en la información anterior se puede concluir que el rendimiento promedio del combustible B es superior al rendimiento del combustible A? Utilice un nivel de significancia del 1%. Los datos son los siguientes: Planta Insecticida A Insecticida B 1 1.3 25. Se desea analizar si existen diferencias en el contenido de proteínas del salmón y el atún enlatados.8 1.3 26. Ejercicio 6. ¿Proporcionan estos datos suficiente evidencia que indique que el diámetro transversal promedio del corazón de los hombres es igual al de las mujeres? Tome un nivel de significancia del 5% Ejercicio 9. Ejercicio 8. Al tiempo se seleccionaron 10 hojas en cada mitad de cada planta fumigada y se registró el número medio de insectos por hoja.2 24.7 26. se fumigó cada planta con ambos productos aplicándolos al azar en cada mitad de la planta.21 1.Sección “B”: 09 12 11 10 10 11 ¿Se puede concluir. 6.4 x1  1. 7.7.4. 8. 1.1 el secado debe continuar. 7. 3. Los resultados fueron: Individuos normales n1=20 S1=0. 7. Se quiso probar si la cirrosis de hígado hacia variar el índice de actividad de la colinesterasa en suero.99.2 La cirrosis de hígado.6. para ello se tomó la siguiente información de una muestra de 11 Gerentes: Consumo anual en miles de $: 36 22 26 18 22 14 34 25 25 18 18 Con base en los resultados anteriores considera que la presunción es cierta. 6. Un agrónomo mide el contenido promedio de humedad en cierta variedad de trigo que fue secado especialmente en una muestra de 16 toneladas: 7.4.2.8 S1=106. Ejercicio 14.8 Individuos cirroticos n2=25 x 2  0.Ratas de control n1=25 x1  869.13. 7.92.98. 3.7 Ratas desnutridas n2=36 x 2  465 S2=153.3. 7. Basados en estos datos ¿podemos decir que la longitud media de los pedazos de cable es mayor de 4 cm? Suponga población normal y tome el nivel de significancia 0. 6.4. los resultados fueron 45 67 54 41 38 59 48 31 59 31 Con esta información se puede concluir que el rendimiento promedio del combustible es superior a 50 km/galón? Utilice un nivel de significancia del 1%. 3.12.37.01. 7. Los siguientes datos corresponden a la longitud medida en centímetros de 18 pedazos de cable sobrantes en cada rollo utilizado: 9.97.66 S2=0. ¿hace variar el índice de la colinesterasa en suero? Ejercicio 11. Si el promedio de humedad excede de 7. 7.04. 2. 7.7 ¿Se observan diferencias significativas entre el grupo control y el grupo desnutrido? Ejercicio 10.05. 5. 4. Se quiere medir el rendimiento de un automóvil.05.3.15. 5.3.86.9.5. Se eligieron dos muestras aleatorias e independientes de individuos. 7. 1. Se presume que el consumo de gerentes por año es igual a 20 mil dólares. 6. de acuerdo con esta evidencia? Tome un nivel de significancia del 5%. 6. 7.1. 3. para ello se determinó el número de km por galón de gasolina.01.87. 6.56. 7.2. 7. ¿Debería continuarse con el proceso de secado. 7.8. 7.24. Ejercicio 12. 4. .41. Ejercicio 15.3. Las muestras deben tener distribuciones normales Prueba estadísticas Sd  d 2 i  nd n 1 2 d  d n t i d D Sd n Intervalo de Confianza. Un ejemplo de observaciones apareadas consiste en considerar a un conjunto de n personas a las que se le aplica un tratamiento y se mide por ejemplo el nivel de estrés antes X 1 y después del tratamiento X2.2 SUPUESTOS 1.5 PRUEBAS DE HIPÓTESIS PARA MEDIAS: MUESTRAS DEPENDIENTES 9.1 INTRODUCCIÓN Las muestras dependientes o apareadas aparecen como distintas observaciones realizadas sobre los mismos individuos. Dos muestras son dependientes cuando se obtienen de sujetos comunes. la producción de una maquina en el turno mañana x1 y la producción de la misma máquina en el turno tarde x2. Las hipótesis a considerar: Hipótesis Caso I Caso II Caso III Ho: μd ≤ D Ho: μd = D Ho: μd ≥ D H 1 : μd > H 1 : D D μd ≠ H 1 : μd < D 3.5. También se denominan muestras apareadas o equiparadas. En las pruebas de datos pareados no hay necesidad de suponer que las dos poblaciones de que se trata tienen varianzas iguales. Las observaciones de las muestras son aleatorios 2.9. Del 100(1-)% para la diferencia de medias poblacionales d es de la forma: .5. sino una sola. 8331. Solución: Obsérvese que las mediciones se realizan sobre las mismas personas. tenemos: .9) = 1. Los resultados se muestran a continuación.05.3 EJERCICIOS RESUELTOS Ejercicio 1. Para ello se realiza un estudio con una muestra aleatoria simple de 10 personas. sd  n  d  t / 2.5. n 1   d  d  t / 2. en la cual lo que nos interesa es la diferencia producida entre el colesterol antes del tratamiento y después del mismo. que se realiza sobre la variable diferencia.n 1 sd   n n x Donde: µd : Media de las diferencias poblacional d : Media de la diferencia de los datos de las muestras apareadas Sd : Desviación estándar de las diferencias de los datos de las muestras apareadas n : Número de pares de datos 9. Se pretende demostrar que cierto tratamiento practicado durante un mes. Para ello introducimos una nueva variable que expresa la diferencia existente entre el colesterol antes del tratamiento y después del mismo: D = Xant − Xdes Antes Después Diferenci a 200 150 50 210 200 10 330 275 55 240 250 -10 260 200 60 300 250 50 245 200 45 210 180 30 190 190 0 225 205 20 Encontrar evidencia a favor de que el tratamiento surgen el efecto deseado (baja el colesterol) es lo mismo que encontrar evidencia estadísticamente significativa en el contraste: Ho: μd = 0 t d  d Sd n H1 : μd > 0 Esto es de nuevo un contraste para una media. ayuda a reducir el colesterol. Antes 200 210 330 240 260 300 245 210 190 225 Despué 150 200 275 250 200 250 200 180 190 205 s ¿Que podemos concluir de estos datos. El estadístico que usamos es: De tablas tenemos que: t(0. por tanto no tenemos dos muestras aleatorias independientes. para un α=5% y n-1=9 grados de libertad Luego si suponemos que la hipótesis nula es cierta y que la variable diferencia sigue una distribución normal de parámetros desconocidos. 019 Despise 10 210..8104 SE Mean 14.05) entonces se rechaza Ho.05.8457 95% lower bound for mean difference: 16.856 7.95 24.0000 24.236 11.000 37.81 .6179 T-Test of mean difference = 0 (vs > 0): T-Value = 3.000 45.846 de rechazo Ho 10 de AceptaciónZona Zona Ho t(0.002 De aquí concluimos que (pvalue=0.491 Difference 10 31. Después Paired T for Antes . t ( d   d ) (31  0)   3.95 P-Value = 0.Despise N Mean StDev Antes 10 241.d  50  10  55  10  .  0  20 10 Sd   31 d 2  nd n 1 2  24 .002)<(α=0..8331 t=3. En MINITAB Ejecutamos las opciones Stat/Basic statistics/ Paired t.9)=1. . luego completamos los datos como se ve en la figura y clickeamos ok.9 Conclusión: El valor experimental se encuentra claramente en la región de rechazo por tanto concluimos que existe evidencia estadísticamente significativa en contra de la hipótesis nula (se rechaza) y a favor de la hipótesis alternativa (al menos con un nivel de significación del 5%). ok Se obtiene los siguientes resultados Paired T-Test and CI: Antes...81 7. 5 -0. A 10.4 11.2 11.1 8. Para hacer la comparación.5  ..1 11.8 8.04 20 n Hipótesis Ho: μd = 0 (no hay diferencia) H1: μd ≠ 0 (hay diferencia) De la tabla t de estuden para dos colas.8)  4.9 12.6 9.9 10.2 9.Ejercicio 2.4  .025) t(0..7 -2.5 -0.  ( 1.2 12.24 d .4 0.3 9.1 9.4 11.7 )  ( 1.6 8.1 11 11.2 10.3 0.3 9.1 11 11.9 9 12.8 9.5  .7)  ( 1.8 12.8 11 9.093 Como (t(0.3 11 Presentan los datos suficiente evidencia para concluir que hay diferencia en el desgaste promedio de los dos tipos de neumáticos? Solución A B d d  10.  0. Un fabricante deseaba comparar la resistencia al desgaste de dos tipos distintos de neumáticos A y B. 2 2 2 2 2  0. para un =0.1 8.7 )  ( 1.64 Sd 1.1 0.6   4.3  ( 1.4 11.1 11.2 12.5 0.3 9.4  0.9 10.9 9 9..8 8.6)  20 ( 4.8)  t d 2  nd n 1 2  ( 21 ..3 9.1 8.8 10 9.7 -1.4  0.5 1.5 8.2 12.2 0.3 10.3 10.4  0..1 1 1.2 1.3 10.64) se rechaza la Ho.2 10. Estos valores aparecen en la siguiente tabla. se asignó al azar un neumático del tipo A y uno del tipo B a las ruedas posteriores de 20 automóviles.7 8.025.5 10. Luego hay suficiente evidencia que hay diferencia en el desgaste de los neumáticos.19)= 2.4 11.2 11 8.  ( 1.1 10 10.6 10.3 8.1 0.9 12.7 0.04 d   d 21.5 8.3 =0.1 10 10.8 9.1 8.4 0.05 (/2=0.2 11  1.8 12.3 8.2 11.7 21. .1 9.6 Sd  9.6 9.5 10.4  0.2 B 10.3 10.. d 2 1 1.4  0.2 9.7 8.8 10 9.7 ) 2 19 -1.3 10.19)= 2.8  d  0.6 0.093 < t=4.8) 20 0  0.3 10.025. Los autos recorrieron un número específico de kilómetros y se observó el desgaste de cada neumático.8 9. en la cual lo que nos interesa es la diferencia producida entre el colesterol antes del tratamiento y después del mismo. luego hay diferencia de rendimientos. B Paired T for A . Solución Hipótesis HO : 1= 2 (No hay diferencia de rendimiento) H1 : 1≠ 2 (hay diferencia de rendimiento) Aplicando minitab.05).B N Mean StDev SE Mean A 9 28. se tiene que (p–value = 0.Ejercicio 3.024 < 0.8 6 29 30 7 27 30.0 . entonces rechazamos Ho.31261) T-Test of mean difference = 0 (vs not = 0): T-Value = -2.42073. por tanto no tenemos dos muestras aleatorias independientes. Paired T-Test and CI: A. este asignó al azar un galón de la estación A y un galón de la estación B a 9 automóviles.2 Si las poblaciones son normales.77 P-Value = 0.2570 Difference 9 -1. Para ello se realiza un estudio con una muestra aleatoria simple de 10 personas.86667 2. ayuda a incrementar el rendimiento académico. para esto contrataron a un Ingeniero que haga el estudio.7710 0. Antes 15 Después 15 11 13 12 14 10 13 8 11 13 14 9 12 5 11 16 17 14 14 ¿Que podemos concluir de estos datos. Un grupo de empresarios desean conocer la diferencia que hay entre el rendimiento de un galón de gasolina del mismo octanaje entre dos establecimientos de servicio A y B. presentan los datos suficiente evidencia para concluir que hay diferencia en el rendimiento de la gasolina según establecimiento.024 Según los datos. Estos valores aparecen en la siguiente tabla Auto Est A Est B 1 32 31 2 28 32.2 3 29. Se pretende demostrar que cierto tratamiento practicado durante un mes a un grupo de alumnos de la FIIS. Solución: Obsérvese que las mediciones se realizan sobre las mismas personas. Los autos recorrieron a una determinada velocidad hasta consumir totalmente el galón de gasolina.1 5 27 30.5711 B 9 30.2 8 31 30.7133 0.67392 95% CI for mean difference: (-3. Los resultados se muestran a continuación. Ejercicio 4.8222 0. sino una sola.9556 1.9 31. -0.02176 0. Para ello introducimos una nueva variable que expresa la diferencia existente entre el rendimiento académico antes del tratamiento y después del mismo: D = Xant − Xdes Antes 15 11 12 10 8 13 9 5 16 14 9 28.7 30.2 4 28 31. en la cual lo que nos interesa es la diferencia producida entre la enseñanza antes de la capacitación y después del mismo.1  0)   3.Después Diferencia 15 0 13 -2 14 -2 13 -3 11 -3 14 -1 12 -3 11 -6 17 -1 14 0 Encontrar evidencia a favor de que el tratamiento incremente el rendimiento es lo mismo que encontrar evidencia estadísticamente significativa en el contraste: Ho: μd = 0 (el tratamiento no incremente el rendimiento) H1 : μd < 0 (el tratamiento incremente el rendimiento) Esto es de nuevo un contraste para una media. por tanto no tenemos dos muestras aleatorias independientes. tenemos: d  0  ( 2)  ( 2)  ( 3)  . ayuda a incrementar el nivel de enseñanza. Ejercicio 5.05. Se pretende demostrar que la capacitación a un grupo de docentes de la FIIS.70 t(0. Solución: Obsérvese que las mediciones se realizan sobre las mismas personas. para un α=5% y n-1=9 grados de libertad Luego si suponemos que la hipótesis nula es cierta y que la variable diferencia sigue una distribución normal de parámetros desconocidos.79 . que se realiza sobre la variable diferencia. Los resultados se muestran a continuación.. Para ello introducimos una nueva variable que expresa la diferencia existente entre el nivel de enseñanza antes de la capacitación y después del mismo: D = Xant − Xdes Antes 12 11 12 15 10 14 9 13 .1 S d  2  d  nd 2 n 1  1.  ( 1)  0 10  2.. El estadístico que usamos es: De tablas tenemos que: t(0.9)=1.79 0.566 de rechazo Ho 10 de AceptaciónZona Zona Ho t=-3. sino una sola.05. t ( d   d ) ( 2. Para ello se realiza un estudio con una muestra aleatoria simple de 8 docentes. 8331.9) = 1.70 1.8331 Conclusión: El valor experimental se encuentra claramente en la región de aceptación por tanto concluimos que existe evidencia estadísticamente significativa a favor de la hipótesis nula. Antes Después 12 13 11 12 12 12 15 14 10 13 14 14 9 12 13 15 ¿Que podemos concluir de estos datos. 05.125 2  d  nd n 1 2  1.. para un α=5% y n-1=7 grados de libertad Luego si suponemos que la hipótesis nula es cierta y que la variable diferencia sigue una distribución normal de parámetros desconocidos.9) = 1. Una empresa consultora realiza pruebas del rendimiento de un determinado software según el uso por horas a la semana que se le da en dos empresas diferentes.Después Diferencia 13 -1 12 -1 12 0 14 13 1 -3 14 0 12 -3 15 -2 Encontrar evidencia a favor de que la capacitación incrementa el nivel de enseñanza es lo mismo que encontrar evidencia estadísticamente significativa en el contraste: Ho: μd = 0 H 1 : μd < 0 Esto es de nuevo un contraste para una media.7)=1. tenemos: d   1  ( 1)  (0)  (1)  .  ( 3)  ( 2) 8 Sd   1.515 Zona de rechazo Ho Zona de Aceptación Ho 8 t=-2.183 1. t ( d   d ) ( 1.457 0. Ejercicio 6.125  0)   2. El estadístico que usamos es: De tablas tenemos que: t(0.183 t(0.05 de nivel de significancia. Se obtiene una muestra aleatoria de ocho semanas con la siguiente información. que se realiza sobre la variable diferencia. 8331.05.895 Conclusión: El valor experimental se encuentra claramente en la región de aceptación por tanto concluimos que existe evidencia estadísticamente significativa a favor de la hipótesis nula. Dia Empresa A Empresa B 1 80 82 2 90 87 3 88 89 4 90 90 5 80 95 6 87 88 7 90 90 8 80 90 Para 0.. ¿puede la empresa consultora concluir que existe una diferencia en el rendimiento de uso del software? Solución: Empresa A Empresa B 80 82 90 87 88 89 90 90 80 95 87 88 90 90 80 90 .457 . 025. Luego no hay diferencia en el rendimiento de uso. Ejercicio 7. antes y después de recibir cierta preparación intensiva en dicho herramienta.365 < t= -1.25 0 -15 -1 0 -10 d  0 n=8 Sd  d 2  nd n 1 2  340  8( 3.365 Como (t(0.365) se acepta la Ho.NET “ORCAS”.04 Hipótesis: d  0 H0: d  0 H1: (No hay diferencia) (Hay Diferencia) t d   d  3.025) t(0.25 ) 2 8 1  6.7)=  2.7)= 2.05 (/2=0. Se sometieron a 10 personas a un examen de programación realizado bajo Visual Basic .52 Sd 6.7)= -2.025. Solución: Antes Después d 70 115 -45 84 148 -64  d  704  d n = 10 88 176 -88 2 110 191 -81 105 158 -53  51174 d  70 .52 < t(0.d -2  d  26  d 3 2 -1  340 d  3.025.25  0   1. para un =0.4 100 178 -78 d  0 110 179 -69 67 140 -73 79 161 -82 86 157 -71 .04 8 n De la tabla t de Estudent para dos colas. los resultados fueron como sigue: Individuo Antes Después 1 70 115 2 84 148 3 88 176 4 110 191 5 105 158 6 100 178 7 110 179 8 67 140 9 79 161 10 86 157 ¿Proporcionan estos datos evidencia suficiente para decir que la preparación fue efectiva? Tome un nivel de significancia del 1%. 01.821 Como (t= -16.38 Hipótesis: d  0 H0: d  0 H1: (No es efectiva) (Si es efectiva) t d   D  70.01 t(0.9)= -2. cada tienda presentó una lista de 8 marcas conocidas de computadoras con sus respectivos precios.4  0   16. El área de logística de la UNHEVAL está realizando cotizaciones para la compra de nuevas computadoras. para esta licitación se presentaron 2 tiendas comerciales locales.Sd  d 2  nd 2 n 1  51174  10 ( 70 .9)= -2.4) 10  1 2  13 . que se muestran a continuación: Marca HP IBM ACER SONY TOSHIBA INTEL LEXMARK MAXTOR Tienda 1 420 560 450 480 370 450 410 460 Tienda 2 400 520 430 480 320 480 390 500 Diferencia 20 40 20 0 50 -30 20 -40 Para 0. ¿puede el área de logística concluir que existe una diferencia en los costos? Solución: d  20  40  20  0  50  30  20  40  10 8 .63 SD 13.01.63 < t(0.38 10 n De la tabla t de Estudent.821) se rechaza la Ho. para un =0.05 de nivel de significancia. Luego hay suficiente evidencia para decir que la preparación fue efectiva. Ejercicio 8. Se sometieron a 10 personas a un test antes y después de recibir cierta instrucción los resultados fueron como sigue: Individuo 1 2 3 4 5 6 7 8 9 Antes 70 84 88 110 105 100 110 67 79 Después 115 148 176 191 158 178 179 140 161 ¿Proporcionan estos datos evidencia suficiente para decir que la instrucción fue efectiva? Tome un nivel de significancia del 1%. Se obtiene una muestra aleatoria de ocho días con la siguiente información.05 de nivel de significancia.61 n 8 Zona de rechazo Ho Zona de Aceptación Ho t =1.4 EJERCICIOS PROPUESTOS Ejercicio 1.860 Conclusión: El valor experimental se encuentra aceptación por tanto concluimos aceptamos la H 0 que no existe diferencia en los costos. obtiene la gasolina en el grifo A y en el grifo B. Una empresa compara el rendimiento de una unidad de gasolina con su auto (km/un).25 t(0.61 d   D 10  0   1.25 SD 22. Los datos fueron los siguientes: Placebo 6 8 4 8 5 6 5 6 4 Tratamiento 5 6 4 5 3 6 6 2 2 10 86 157 5 6 . un médico ajeno al proyecto evaluó la importancia del eczema en una escala de 0 (no eczema) a 10 (tamaño máximo de eczema). 9. Dia Grifo A Grifo B 1 80 82 2 90 87 3 88 89 4 90 90 5 80 95 6 87 88 7 90 90 8 80 90 Para 0. se tomaron 10 pacientes con eczema de más de 9 meses y se les sometió durante 3 semanas a un tratamiento ficticio (placebo) y durante las tres siguientes a un tratamiento con ácidos grasos.05.5. ¿puede la empresa concluir que existe una diferencia en el rendimiento de gasolina según los grifos? Ejercicio 2. Ejercicio 3. Para comprobar si un tratamiento con ácidos grasos es eficaz en pacientes con eczema atípico.8)=1.Sd  t d 2  nd 2  n 1 4360  8(10 ) 2 8 1  22 . Tras cada periodo. 05. 15 pacientes hipertensos son sometidos al programa y controlados en su tensión asistólica antes y después de 6 meses de tratamiento. Los datos son los siguientes: Individuo Fin 180 140 200 170 160 160 170 140 180 130 190 150 190 140 180 150 190 190 160 170 170 120 190 160 200 170 210 160 220 150 ¿Es efectivo el tratamiento? Ejercicio 5. Alumno Profesor A Profesor B 1 2 3 4 5 6 7 8 9 10 11 12 13 14 13 14 18 14 17 15 16 17 13 11 15 15 18 16 14 13 17 15 16 17 17 18 14 12 18 15 20 15 Ejercicio 8. A una empresa le ofrecen impartir un curso de capacitación para aumentar el rendimiento de sus trabajadores. Placebo H-cloro 211 181 210 172 210 196 203 191 196 167 190 161 191 178 177 160 173 149 170 119 163 156 Según estos datos experimentales. Para eso se hace calificar a cada profesor. Una sala en silencio y una sala con música ambiente. a) Dos grupos de 7 cerdos fueron alimentados durante un cierto número de días con dos alimentos A y B. Alimento A Alimento B 33 31 53 47 34 33 29 30 39 39 41 38 b) Supongamos ahora que el peso inicial de los animales es un factor que podría incidir en los aumentos de peso y por ello se consideró que las raciones se aplicaran a parejas de cerdos de igual peso. Sometieron a 10 estudiantes a un test de aptitud matemática bajo dos condiciones. En un programa de Control de Enfermedades Crónicas. los exámenes escritos de 14 alumnos. ¿podemos afirmar que existe diferencia en la presión sistólica media durante la utilización de estos dos fármacos? Ejercicio 6. se controla el tiempo que tardaban estos 31 29 . separadamente. Los puntajes obtenidos fueron. Use nivel de significación 0. volvamos a realizar la prueba que corresponde a esta nueva situación para ver si los alimentos son equivalentes o no. Suponiendo ahora que los datos que aparecen en la tabla corresponden a datos apareados. Determine si existe diferente criterio entre los profesores para colocar la nota. Se desea saber si existe diferencia de criterio entre dos profesores. Con los datos de incremento de pesos adjuntos. respecto a la calificación de un examen escrito. La tabla siguiente muestra los efectos de un placebo y de la hidroclorotiacida sobre la presión sanguínea sistólica de 11 pacientes. Las calificaciones aparecen en la tabla. Estudiantes 1 2 3 4 5 6 7 8 9 10 Ptje sin música 114 121 136 102 99 114 127 150 129 130 Ptje con música 112 122 141 107 96 109 121 146 127 128 ¿Hay diferencias en los puntajes? Ejercicio 9.¿Es eficaz el tratamiento? Ejercicio 4. Ejercicio 7. pruebe la hipótesis de que los alimentos son equivalentes. La empresa decide enviar a quince de sus trabajadores elegidos al azar de toda la plantilla. Para comprobar si el curso es beneficioso. la hipertensión Está incluida como la primera patología a controlar. 6 20.0 18.0 19.trabajadores en realizar un trabajo antes de realizar el curso y después de realizar el curso.4 21.6 22. Los pesos fueron (en kilogramos): Antes Después 85 73 70 51 54 56 56 73 72 125 103 87 88 85 77 75 76 60 69 46 Dentro de un nivel de confianza del 99%.0 20. Como esta está muy correlacionada con un correcto peso corporal.3 16.5 18. al comenzar el primer año y al finalizar. considere las muestras independientes? 45 107 91 87 .3 16.0 20.0 20. Los resultados se reflejan en la tabla adjunta.8 23. En una universidad.3 21.0 23.0 18. Trab Ant Desp 1 23.5 21.0 22.0 15.0 22.1 18.4 15.5 12.3 18.1 18. la encargada de bienestar social piensa que la responsabilidad por los estudios de los estudiantes de primer año hace que disminuyan la actividad física.6 18. tomó una muestra de los pesos de 12 estudiantes.4 21.0 Ejercicio 10.4 2 3 4 5 6 7 8 9 10 11 12 13 14 15 12. ¿puede afirmarse que ha aumentado el peso corporal de los estudiantes durante el primer año.1 12.5 17.
Copyright © 2024 DOKUMEN.SITE Inc.